You are on page 1of 55

Syllabus for Math 233 Section 8 Fall 2014

See Web-site: www.math.umass.edu/bill/m233/233-F14.html for further information on the course, on reviews and on the exams. It will updated throughout the
course.
Instructor: William Meeks, 1536 LGRT
Email: bill@math.umass.edu, office phone 545-4239
Office Hours: By appointment and on the days and hours I am in the calculus help
center, which are Tuesdays from 11:00 to 12:00. Normally I will have all day office
hours in the Blue Wall on the days of the exams and many students find these office
hours are extremely useful to them. Also there will probably be an undergraduate
TA associated to this course and he/she might also be available to help you some to
in his/her office hours.
Reviews for exams: I will be giving 1 review for each exam just for our section on
the night before each midterm; these reviews for exams also seem to be very helpful
to the students in this class.
Textbook: Calculus: Early Transcendentals 7e (7-th Edition customized hybrid
version for Umass at Amherst) by James Stewart
Grading: Exam 1, Exam 2 and Final Exam each count 25% of your grade and
25% of your grade comes from your section grade. All scores will be scaled to a
0-100 scale before averaging. Roughly, the section grade will be 15% WebAssign
homework and 10% in class exercises to be turned in to me in class; these exercises
appear throughout the lecture note handout that is given to you on the first day
of class. See the web pages and course syllabus on the course web page for further
information on grading and exams.

BASICS.

Study guide for Exams 1 and 2 and 3.


by William H. Meeks III September 1, 2014
1

Basics.

First we cover the basic definitions and then we go over related problems. Note that
the material for the actual midterm may include material from the review guide
for midterm 2. Before the exam, view the updated course web page for the exact
material covered on midterm 1.
Definition 1 Let n be a positive integer. Then the cartesian product of n copies
of the real number line R is:

Rn = R R . . . R = {(a1 , a2 , . . . , an | aj R)},
which is the set of all ordered n-tuples of real numbers.
Example 2

(a) R2 = R R = {(a1 , a2 ) | ai R} is the Euclidean plane.

(b) R3 = R R R = {(a1 , a2 , a3 ) | ai R3 } is Euclidian three-space.


There are two standard notations for points in R3 , or more generally Rn . If
P R3 , then P = (a1 , a2 , a3 ) for some scalars a1 , a2 , a3 . The book also denotes this
point by writing P (a1 , a2 , a3 ). The scalar a1 is called the x-coordinate of P , a2 is
called the y-coordinate of P and a3 is called the z-coordinate of P .
Example 3 The point P = (1, 0, 7) in R3 can also be written as P (1, 0, 7). Its
z-coordinate is 7.
Definition 4

(a) Given points P = (x1 , y1 , z1 ) and Q = (x2 , y2 , z2 ) in R3 , then

P Q = hx2 x1 , y2 y1 , z2 z1 i denotes the arrow or vector based at P with


terminal point Q.
(b) If R is a scalar and v = ha, b, ci is a vector, then consider the new vector
v = ha, b, ci; if > 0, then v is the vector pointed in the direction v
and has length |v|; if < 0, then v is the vector pointed in the opposite
direction of v with length |||v|.
(c) If u = hx1 , y1 , z1 i and v = hx2 , y2 , z2 i, then u + v = hx1 + x2 , y1 + y2 , z1 + z2 i.
In other words, vectors add by adding their coordinates.
Definition 5 If a = hx1 , y1 , z1 i and b = hx2 , y2 , z2 i, then the dot product of a
and b is:
a b = x1 x2 + y1 y2 + z1 z2 .
Example 6 The dot product of h1, 2, 3i and h1, 0, 7i is
h1, 2, 3i h1, 0, 7i = 1 + 0 + 21 = 22.
It turns out that the length of a vector can be found by using the dot product
and it satisfies some nice algebraic properties listed in the next two theorems.

BASICS.

Theorem 7 Let a = hx1 , y1 , z1 i be vector and let P = (x2 , y2 , z2 ), Q = (x3 , y3 , z3 )


be points. Then:
p

1. The length of a is |a| = a a = x21 + y12 + z12 .


2. The distance d(P, Q) from the point P to the point Q is:

d(P, Q) = |P Q| =

p
(x3 x2 )2 + (y3 y2 )2 + (z3 z2 )2

Theorem 8 (Basic algebraic properties of dot product) Let a = hx1 , y1 , z1 i,


b = hx2 , y2 , z2 i, c = hx3 , y3 , z3 i be vectors and let be a scalar.
1. a b = b a.
2. (a + b) c = a c + b c.
3. (a) b = (a b).
Definition 9 The sphere in R3 with center C = (x0 , y0 , z0 ) and radius r is the set
where (x x0 )2 + (y y0 )2 + (z z0 )2 = r2 . Note that this sphere is geometrically
the set of points (x, y, z) of distance r from the point (x0 , y0 , z0 ).
Example 10 Consider the subset of R3 defined by x2 + y 2 + 6y + z 2 + 2z = 26. By
completing the square, we have
x2 + (y 2 + 6y + 9) + (z 2 + 2z + 1) = 26 + 9 + 1 = 36,
which simplifies to be
x2 + (y + 3)2 + (z + 1)2 = 62 .
So this set is the sphere centered at (0, 3, 1) of radius 6.
For convenience, it is useful to pick out the special unit vectors pointed respectively
along the positive x, y and z-axes, as given in the next definition.
Definition 11 We define the standard basis vectors for R3 as follows: i = h1, 0, 0i,
j = h0, 1, 0i, k = h0, 0, 1i. Note that the vector ha, b, ci can be expressed by ha, b, ci =
ai + bj + ck.
For nonzero vectors a, b
a b = |a||b| cos(),
where [0, ] is the angle between the vectors. It follows that:
1. a and b are orthogonal or perpendicular if and only if a b = 0.
2. The angle between a and b is an acute angle if and only if a b > 0
3. The angle between a and b is an obtuse angle if and only if a b < 0.
4. cos() =

ab
|a||b| .

ab
ab
5. = arccos( |a||b|
) = cos1 ( |a||b|
) In particular, if a and b are unit vectors, then
= arccos(a b).

BASICS.

Example 12 The vectors h1, 2, 1i and h3, 1, 1i are orthogonal, since h1, 2, 1i
h3, 1, 1i = 3 2 1 = 0.
Definition 13
1. The scalar projection (component) of b onto a is compa b =
ab
|a| . In particular, if a is a unit vector, then compa b = a b.
2. The vector projection of b onto (in the direction of) a is a proja b = ( ab
aa )a.
In particular, if a is a unit vector, then proja b = (a b)a.
3. The direction cosines of the vector b are:
(a) cos() =

b
|b|

i,

(b) cos() =

b
|b|

j,

(c) cos() =

b
|b|

k,

and so, , , are the respective angles that b makes with the x, y and z-axes.
Example 14 Consider the vectors a = h1, 2, 2i and b = h1, 1, 1i. Since proja b =
ab
aa a, then
5
4 1 1
v = b proja b = h1, 1, 1i h1, 2, 2i = h , , i
9
9 9 9
must be perpendicular to a and must lie in the plane containing a and b.
Definition 15

1. The determinant of the matrix M with


rows vectors v =
a b
= ad bc.
ha, bi and w = hc, di can be calculated by: |M | =
c d

The absolute value |ad bc| of this determinant equals the area of the parallelogram with sides v and w.
2. The determinant of the matrix M with rows vectors a = ha1 , a2 , a3 i, b =
hb1 , b2 , b3 i and c = hc1 , c2 , c3 i can be calculated by:


a1 a2 a3








b2 b3
b1 b3
b1 b2








|M | = b1 b2 b3 = a1
a2
+ a3
c2 c3
c1 c3
c1 c2
c1 c2 c3
The absolute value of the determinant |M | equals the volume of the parallelepiped or box spanned by the vectors a, b and c.
3. The cross product a b
be calculated by:

i
j k

a b = a1 a2 a3
b1 b2 b3

of vectors a = ha1 , a2 , a3 i and b = hb1 , b2 , b3 i can













a2 a3
=
i a1 a3 j + a1 a2 k.
b1 b3
b1 b2
b2 b3

The length of a b is given by: |a b| = |a||b| sin(), where [0, ] is the


angle between a and b. Also |a b| is area of the parallelogram with sides a
and b. Note that it follows that area of the triangle with vertices h0, 0, 0i and
the position vectors a and b is |ab|
2 .

BASICS.

Example 16 Consider the points A = (1, 0, 1), B = (0, 2, 3) and C = (1, 1, 0).
Then the area of the triangle 4 with these vertices can be found by taking the area

of the parallelogram spanned by AB and AC and dividing by 2. Thus:



i
j
k


|AB AC|
1
1
1
1
2 = |h0, 5, 5i| =
Area(4) =
0 + 25 + 25 =
50
= 1 2
2
2
2
2
2


2 1 1
Example 17 Consider the vectors a = h1, 0, 1i, b = h0, 2, 3i and c = h1, 7, 0i.
Then the volume of the parallelepiped or box spanned by these 3 vectors is:


1 0 1


0 2 3 = | 21 0 + 2| = | 19| = 19


1 7 0
a
Definition 18 If F is a force with magnitude A applied in the unit direction |a|
to
an object in order to move it from the point P to the point Q, then the work W

done is: W =

A
|a| a

P Q.

Example 19 If F is a force of 10N (10 Newtons) applied in the unit direction


1 h2, 1, 1i to an object to move it from P = (3, 2, 5) to Q = (1, 2, 3), then the
6
work done is (length measured in meters):
100N m
10N
,
W = h2, 1, 1i h4, 4, 2i =
6
6
where m is one meter.
Definition 20 The torque on a rigid body with position vector a with a force of
b
magnitude A in the unit direction |b|
is:
=aA

b
.
|b|

Example 21 What is the magnitude (the length) of the torque on a rigid body with
b
position vector a = h1, 1, 3i with a force of 10N in the direction of |b|
= 16 h2, 1, 1i
(length measured in meters m) ?
Solution :
10N m
10N m
10N m

| | = |h1, 1, 3i h2, 1, 1i| = | h4, 5, 3i| =


6
6
6

50

Definition 22 Given a point P = (x0 , y0 , z0 ) and a vector v = ha, b, ci, the vector
equation of the line L passing through P in the direction of v is:
r(t) = P + tv = hx0 , y0 , z0 i + tha, b, ci = hx0 + at, y0 + bt, z0 + cti.
The resulting equations:
x = x0 + at,
y = y0 + bt,
z = z0 + ct,

BASICS.

are called the parametric equations for L. The resulting equations (solving for
t):
x x0
y y0
z z0
=
=
,
a
b
c
are called the symmetric equations for L.
Example 23 The vector equation for the line L passing through P = (1, 2, 3) and
Q = h4, 0, 7i is given by:

r(t) = P + tP Q = h1, 2, 3i + th3, 2, 4i = h1 + 3t, 2 2t, 3 + 4ti.


Definition 24 The plane passing through the point P = (x0 , y0 , z0 ) with normal
vector n = ha, b, ci is given by the following equation, where (x, y, z) denotes a
general point on the plane:
0 = n hx x0 , y y0 , z z0 i.
Equivalently, we have:
a(x x0 ) + b(y y0 ) + c(z z0 ) = 0.
Example 25 The equation of the plane passing through P = (1, 2, 3) and with
normal vector n = h3, 4, 1i is:
3(x 1) + 4(y 2) + (z 3) = 0.
Example 26 Find the equation of the plane passing through points P = (1, 0, 2), Q =
(4, 2, 3), R = (2, 0, 4).
Solution : Since a plane is determined by its normal vector n and a point on it,
say the point P , it suffices to find n. Note that:


i j k



n = P Q P R = 3 2 1 = h4, 5, 2i.
1 0 2
So the equation of the plane is:
4(x 1) 5y 2(z 2) = 0.

Given two planes with unit normal vectors n1 and n2 , respectively, then the cosine of the angle between them is the cosine of the angle between the lines determined
by n1 and n2 , which can be calculated using dot products.
Example 27 The cosine of the angle between x2y +2z = 1 and 2xy +2z = 10
is given by
1
1
1
8
cos() = | h1, 2, 2i h2, 1, 2i| = (2 + 2 + 4) = .
3
3
9
9
Definition 28 Let r(t) be a vector valued curve in R3 , where r(t) = hf (t), g(t), h(t)i.
Here t is called the parameter of r(t). If the derivative r0 (t) = limh0 r(t+h)r(t)
h
exists for each t, then the curve r(t) is called differentiable and r0 (t) is called the
derivative or velocity or tangent vector field v(t) = r0 (t) to the curve r(t). The
length |v(t)| is called the speed of the curve r at the parameter value t.

SOME PRACTICE PROBLEMS SOLVED.

Theorem 29 If r(t) = hf (t), g(t), h(t)i is a differential curve in R3 , then:


r0 (t) = hf 0 (t), g 0 (t), h0 (t)i.
Conversely, if f (t), g(t), h(t) are differentiable functions, then r(t) is differentiable.
The speed function for r(t) is then:
p
speed(t) = (f 0 (t))2 + (g 0 (t))2 + (h0 (t))2 .
Example 30 Suppose r(t) = ht, sin(2t), t2 + 1i, then r0 (t) = h1, 2 cos(2t), 2ti with
r0 (0) = h1, 2, 0i. Hence, the tangent line to r(t) at t = 0 is given by:
L(t) = r(0) + tr0 (0) = h1, 0, 1i + th1, 2, 0i = h1 + t, 2t, 1i
p
and the speed function of r(t) is: speed(t) = 12 + 4 cos2 (2t) + 4t2 .
Definition 31 The length L of a parameterized curve r(t) in R3 on a time interval[a, b]
is the integral of the speed:
Z b
|r0 (t)|dt.
L=
a

Example 32
2ti, then r0 (t) = hcos(t), sin(t), 2i with conp If r(t) = hsin(t), cos(t),
2
2
stant speed cos (t) + sin (t) + 4 = 5. Hence, the length of r(t) from time t = 1
to time t = 6 is:

Z 6
6

L=
5 dt = 5t = 5 6 5 1 = 5 5.
1

Some practice problems solved.


1. Find parametric equations for the line which contains A(2, 0, 1) and B(1, 1, 1).

Solution : Let v = AB = h1, 1, 1ih2, 0, 1i = h3, 1, 2i. Since A(2, 0, 1)


lies on the line, then:
x = 2 3t,
y = 0 t = t,
z = 1 2t.
2. Determine whether the lines l1 : x = 1 + 2t, y = 3t, z = 2 t and l2 : x =
1 + s, y = 4 + s, z = 1 + 3s are parallel, skew or intersecting.
Solution : Vector part of line l1 is v1 = h2, 3, 1i and for line l2 is v2 =
h1, 1, 3i. Clearly, v1 is not a scalar multiple of v2 and so these lines are not
parallel. If these lines intersect, then for some values of t and s:
x = 1 + 2t = 1 + s 2t = 2 + s,
y = 3t = 4 + s 3t = 4 + s.
Solving these two linear equations yields:
t = 6 and s = 14.
Plugging these values into z = 2 t = 1 + 3s yields the inequality 4 6= 43,
which means there is no solution and the lines do not intersect. Thus, the lines
are skew.

SOME PRACTICE PROBLEMS SOLVED.

3. Find an equation of the plane which contains the points P (1, 2, 1), Q(1, 2, 1)
and R(1, 1, 1).

Solution : Consider the vectors P Q = h2, 4, 0i and P R = h2, 1, 2i which


lie parallel to the plane. Then consider the normal vector:


i j
k


n = P Q P R = 2 4 0 = 8i + 4j + 6k.
2 1 2
So the equation of the plane is given by:
h8, 4, 6i hx + 1, y 2, z 1i = 8(x + 1) + 4(y 2) + 6(z 1) = 0.
4. Find the distance from the point (1, 2, 1) to the plane 2x + y 2z = 1.
Solution : The normal to the plane is n = h2, 1, 2i and the point P =
(0, 1, 0) lies on this plane. Consider the vector from P to (1, 2, 1) which is
v = h1, 1, 1i. The distance from (1, 2, 1) to the plane is equal to:



n
5
1

|compn v| = v
= |h1, 1, 1i h2, 1, 2i| = .

|n|
3
3
5. Let two space curves
r1 (t) = hcos(t 1), t2 1, t4 i, r2 (s) = h1 + ln s, s2 2s + 1, s2 i,
be given where t and s are two independent real parameters. Find the cosine
of the angle between the tangent vectors of the two curves at the intersection
point (1, 0, 1).
Solution : After taking derivatives, we obtain:
r01 (t) = h sin(t 1), 2t, 4t3 i,
1
r02 (s) = h , 2s 2, 2si.
s
At the point (1, 0, 1), t = 1 and s = 1 and so, r01 (1) = h0, 2, 4i and r02 (1) =
h1, 0, 2i are the related tangent vectors. Thus,
cos() =

r01 (1) r02 (1)


8
4
0
= = .
0
|r1 (1)| |r2 (1)|
5
20 5

6. Suppose a particle moving in space has velocity


v(t) = hsin(t), cos(2t), et i
and initial position r(0) = h1, 2, 0i. Find the position vector function r(t).
Rt
Solution : Since r0 (t) = hsin(t), cos(2t), et i, then r(t) =
v(s)ds. Thus,
r(t) = h cos(t) + x0 , 21 sin (2t) + y0 , et + z0 i with r(0) = h1 + x0 , y0 , 1 + z0 i =
h1, 2, 0i. Hence, x0 = 2, y0 = 2, z0 = 1 and so, r(t) = h cos(t)+2, 21 sin (2t)+
2, et 1i.

SOME PRACTICE PROBLEMS SOLVED.

7. Find the center and radius of the sphere x2 + y 2 + z 2 + 6z = 16.


Solution :
equation:

Complete squares to obtain from x2 + y 2 + z 2 + 6z = 16, the


x2 + y 2 + (z + 3)2 = 16 + 9 = 25.

Hence, the center is at C = (0, 0, 3) and the radius is 5.

Review guide for midterm 2 in Math 233


Midterm 2 covers material that begins approximately with the definition of partial derivatives in Chapter 14.3 and ends approximately with methods for calculating
the double integral of a function f (x, y) over a domain D described in the xy-plane.
See the updated course web page for the exact material covered on this exam.
Definition 33 (Partial Derivatives) If f is a function of two variables, its partial derivatives are the functions fx and fy defined by
f (x + h, y) f (x, y)
h0
h

fx (x, y) = lim
fy (x, y) = lim

h0

f (x, y + h) f (x, y)
.
h

We have the following rule for calculating partial derivatives.


1. To find fx , regard y as a constant and differentiate f (x, y) with respect to x.
2. To find fy , regard x as a constant and differentiate f (x, y) with respect to y.
Example 34 Calculate fx , fy for f (x, y) = x2 exy + y 2 .
Solution : We apply the sum, product and chain rules for derivatives, to get:
fx (x, y) = 2xexy + x2 exy y = 2xexy + x2 yexy
fy (x, y) = x2 exy x + 2y = x3 exy + 2y.
Definition 35 (Second Partial Derivatives) For z = f (x, y), we use the following notation:
 
f
2f
2z
(fx )x = fxx =
=
=
x x
x2
x2
 
f
2f
2z
(fx )y = fxy =
=
=
y x
yx
yx
 
f
2f
2z
(fy )x = fyx =
=
=
x y
xy
xy
 
f
2f
2z
=
=
(fy )y = fyy =
y y
y 2
y 2
Example 36 Find the second partial derivatives of
f (x, y) = x3 + x2 y 3 2y 2 .

SOME PRACTICE PROBLEMS SOLVED.

10

Solution : Note:
fx (x, y) = 3x2 + 2xy 3

fy (x, y) = 3x2 y 2 4y.

Therefore,
fxx = 6x + 2y 3
fyx = 6xy 2

fxy = 6xy 2
fyy = 6x2 y 4.

Note that in the above example fxy = fyx . This is no coincidence and follows
from the next theorem that states that under weak conditions on f (x, y), taking
partial derivatives is a commutative process.
Theorem 37 (Clairauts Theorem) Suppose f is defined on a disk D that contains the point (a, b). If the functions fxy and fyx are both continuous on D, then
fxy (a, b) = fyx (a, b).
The next definition of tangent plane generalizes in a natural way the following
equation of the tangent line of a function of 1 variable:
y y0 = f 0 (x0 )(x x0 ).
Definition 38 (Tangent Plane) Suppose f has continuous partial derivatives.
An equation of the tangent plane to the surface z = f (x, y) at the point P =
(x0 , y0 , z0 ) is
z z0 = fx (x0 , y0 )(x x0 ) + fy (x0 , y0 )(y y0 ).
Example 39 Find the tangent plane to the elliptic paraboloid z = 2x2 + y 2 at the
point (1, 1, 3).
Solution : Let f (x, y) = 2x2 + y 2 . Then
fx (x, y) = 4x

fy (x, y) = 2y

fx (1, 1) = 4

fy (1, 1) = 2.

Then Definition 38 gives the equation of the tangent plane at (1, 1, 3) as


z 3 = 4(x 1) + 2(y 1)
or
z = 4x + 2y 3.
The next definition of linear approximation generalizes the linear approximation
L(x) of a function f (x) of 1 variable at a point x0 = a :
L(x) = f (a) + f 0 (x)(x a).
Definition 40 (Linear Approximation) The linear approximation of f (x, y) at
(a, b) is
L(x, y) = f (a, b) + fx (a, b)(x a) + fy (a, b)(y b).

SOME PRACTICE PROBLEMS SOLVED.

11

Example 41 Use the linear approximation L(x, y) to f (x, y) = xexy at (1, 2) to


estimate f (1.1, 1.8).
Solution :
f 1, 2) = e2
fx (x, y) = exy + xyexy
fx (1, 2) = e2 + 2e2

fy (x, y) = x2 exy
fy (1, 2) = e2 .

L(x, y) = f (1, 2) + fx (1, 2)(x 1) + fy (1, 2)(y 2).


Hence,
L(1.1, 1.8) = e2 + (e2 + 2e2 )(.1) + e2 (.2).
Definition 42 If z = f (x, y), then f is differentiable at (a, b) if the change z of
z can be expressed in the form
z = fx (a, b)x + fy (a, b)y + 1 x + 2 y,
where 1 and 2 0 as (x, y) (0, 0).
The next theorem gives a simple condition for f (x, y) to satisfy in order to be
differentiable.
Theorem 43 If the partial derivatives fx and fy exist near (a, b) and are continuous
at (a, b), then f is differentiable at (a, b).
Definition 44 (Total Differential) For z = f (x, y),
dz = fx (x, y) dx + fy (x, y) dy =

z
z
dx +
dy.
x
y

Example 45 The base radius and height of a right circular cone are measured as
10 cm and 25 cm, respectively, with a possible error in measurement of as much as
0.1 cm. in each. Use differentials to estimate the maximum error in the calculated
volume of the cone.
Solution : The volume V of a cone with base radius r and height h is V = r2 h3 .
So the differential of V is
dV =

V
V
2rh
r2
dr +
dh =
dr +
dh.
r
h
3
3

Since each error is at most 0.1 cm, we have |r| 0.1, |h| 0.1. To find the
largest error in the volume we use the largest error in the measurement of r and of
h. Therefore, we take dr = 0.1 and dh = 0.1 along with r = 10, h = 25. This gives
the estimate
500
100
60
dV =
(0.1) +
(0.1) =
= 20.
3
3
3
Thus, the maximum error in the calculated volume is about 20 cm3 63 cm3 .
Example 46 The dimensions of a rectangular box are measured to be 75 cm, 60
cm, and 40 cm, and each measurement is correct to within 0.2 cm. Use differentials
to estimate the largest possible error when the volume of the box is calculated from
these measurements.

SOME PRACTICE PROBLEMS SOLVED.

12

Solution : If the dimensions of the box are x, y, and z, its volume is V = xyz and
so
V
V
V
dV =
dx +
dy +
dz = yz dx + xz dy + xy dz.
x
y
z
We are given that |x| 0.2, |y| 0.2, and |z| 0.2. To find the largest error
in the volume, we use dx = 0.2, dy = 0.2, and dz = 0.2 together with x = 75,
y = 60, and z = 40:
V dV = (60)(40)(0.2) + (75)(40)(0.2) + (75)(60)(0.2) = 1980.
Thus, an error of only 0.2 cm in measuring each dimension could lead to an error
of as much as 1980 cm3 in the calculated volume! This may seem like a large error,
but its only about 1% of the volume of the box.
Theorem 47 (Chain Rule Case 1) Suppose that z = f (x, y) is a differentiable
function of x and y, where x = g(t) and y = h(t) are both differentiable functions
of t. Then z is a differentiable function of t and
dz
f dx f dy
=
+
.
dt
x dt
y dt
Theorem 48 (Chain Rule Case 2) Suppose that z = f (x, y) is a differentiable
function of x and y, where x = g(s, t) and y = h(s, t) are both differentiable functions
of s and t. Then z is a differentiable function of t and
z
z x z y
=
+
s
x s y s

z
z x z y
=
+
.
t
x t
y t

Theorem 49 (The Chain Rule (General Version)) Suppose that u is a differentiable function of n variables x1 , x2 , . . . , xn and each xj is a differentiable function
of the m variables t1 , t2 , . . . , tm . Then u is a function of t1 , t2 , . . . , tm and
u
u x1
u x2
u xn
=
+
+ ... +
ti
x1 ti
x2 ti
xn ti
for each i = 1, 2, . . . , m.
Example 50 If z = x2 y + 3xy 4 , where x = sin 2t and y = cos t, find

dz
dt

when t = 0.

Solution : The Chain Rule gives


dz
f dx f dy
=
+
= (2xy + 3y 4 )(2 cos 2t) + (x2 + 12xy 3 )( sin t).
dt
x dt
y dt
Its not necessary to substitute the expressions for x and y in terms of t. We simply
observe that when t = 0 we have x = sin 0 = 0 and y = cos 0 = 1. Therefore,
dz
dt

= (0 + 3)(2 cos 0) + (0 + 0)( sin 0) = 6.


t=0

Example 51 The pressure P (in kilopascals), volume V (in liters), and temperature
T (in kelvins) of a mole of an ideal gas are related by the equation P V = 8.31T .
Find the rate at which the pressure is changing when the temperature is 300 K and
increasing at a rate of 0.1 K/s and the volume is 100 L and increasing at a rate of
0.2 L/s.

SOME PRACTICE PROBLEMS SOLVED.

13

Solution : If t represents the time elapsed in seconds, then at the given instant
we have T = 300, dT /dt = 0.1, V = 100, dV /dt = 0.2. Since P = 8.31 VT , with
P
8.31
T
8.31T
T = V and V = V 2 , then Case 1 of the Chain Rule gives
dP
P dT
P dV
8.31 dT
8.31T dV
=

dt
T dt
V dt
V dt
V 2 dt
8.31
8.31(300)
(0.1)
(0.2) = 0.04155.
100
1002
The pressure is decreasing at a rate of about 0.042 kPa/s.
=

Example 52 If z = ex sin y, where x = st2 and y = s2 t, find

z
s

and

z
t .

Solution : Applying Case 2 of the Chain Rule, we get


dz
z x z y
=
+
= (ex sin y)(t2 ) + (ex cos y)(2st)
ds
x s y s
2

= t2 est sin(s2 t) + 2stest cos(s2 t),


z x z y
dz
=
+
= (ex sin y)(2st) + (ex cos y)(s2 )
dt
x t
y t
2

= 2srest sin(s2 t) + s2 est cos(s2 t).


Example 53 If u = x4 y + y 2 z 3 , where x = rset , y = rs2 et , and z = r2 s sin yt, find
the value of u/s when r = 2, s = 1, t = 0.
Solution : We have

u
u x u y u z
=
+
+
s
x s
y s
z s

= (4x3 y)(ret ) + (x4 + 2zy 3 )(2rset ) + (3y 2 z 2 )(r2 sin t).


When r = 2, s = 1, and t = 0, we have x = 2, y = 2, and z = 0, so
u
= (64)(2) + (16)(4) + (0)(0) = 192.
s
Theorem 54 (Implicit Differentiation) Suppose that z is given implicitly as a
function z = f (x, y) by an equation F (x, y, z) = 0, i.e., F (x, y, f (x, y)) = 0 for all
(x, y) in the domain of f (x, y). Then:
F
z
x
= F
x
z

Example 55 Find

z
x

and

z
y

z
y
= F .
y
z

if x3 + y 3 + z 3 + 6xyz = 1.

Solution : Let F (x, y, z) = x3 + y 3 + z 3 + 6xyz 1. Then, from Theorem 54, we


have
z
Fx
3x2 + 6yz
x2 + 2yz
=
= 2
= 2
x
Fz
3z + 6xy
z + 2xy
Fy
z
3y 2 + 6xz
y 2 + 2xz
=
= 2
= 2
.
y
Fz
3z + 6xy
z + 2xy

SOME PRACTICE PROBLEMS SOLVED.

14

Definition 56 (Directional Derivative) The directional derivative of f (x, y)


at (x0 , y0 ) in the direction of a unit vector u = ha, bi is
Du f (x0 , y0 ) = lim

h0

f (x0 + ha, y0 + hb) f (x0 , y0 )


h

if this limit exists.


Definition 57 (Directional Derivative) The directional derivative of f (x, y, z)
at (x0 , y0 , z0 ) in the direction of a unit vector u = ha, b, ci is
f (x0 + ha, y0 + hb, z0 + hc) f (x0 , y0 , z0 )
h0
h

Du f (x0 , y0 , z0 ) = lim
if this limit exists.

Definition 58 (Gradient) If f is a function of two variables x and y, then the


gradient of f is the vector function f defined by
f (x, y) = hfx (x, y), fy (x, y)i =

f
f
i+
j.
x
y

Definition 59 (Gradient) For f (x, y, z), a function of three variables,


f = hfx , fy , fz i =

f
f
f
i+
j+
k.
x
y
z

The next two theorems give simple rules for calculating the directional derivative
of a function in 2 or 3 variables in terms of the gradient of the function.
Theorem 60 If f is a differentiable function of x and y, then f has a directional
derivative in the direction of any unit vector u = ha, bi and
Du f (x, y) = fx (x, y)a + fy (x, y)b.
Theorem 61 If f is a differentiable function of x, y, and z, then f has a directional
derivative in the direction of any unit vector u = ha, b, ci and
Du f (x, y, z) = f (x, y, z) u.
By the above two theorems, we have for any unit vector u,
Du f = f u = |f ||u| cos() = |f | cos().
Thus, the next theorem holds.
Theorem 62 Suppose f is a differentiable function of two or three variables. The
maximum value of the directional derivative Du f (x) is |f (x)| and it occurs when
u has the same direction as the gradient vector f (x).
Example 63 Find the directional derivative of the function f (x, y) = x2 y 3 4y at
the point (2, 1) in the direction of the vector v = 2i + 5j.

SOME PRACTICE PROBLEMS SOLVED.

15

Solution : We first compute the gradient vector at (2, 1):


f (x, y) = 2xy 3 i + (3x2 y 2 4)j
f (2, 1) = 4i + 8j.

Note that v is not a unit vector, but since |v| = 29, the unit vector in the direction
of v is
v
2
5
u=
= i + j.
|v|
29
29
Therefore, by Theorem 60, we have
2
5
Du f (2, 1) = f (2, 1) u = (4i + 8j) ( i + j)
29
29
=

32
4 2 + 8 5

= .
29
29

Theorem 64 Suppose S is a surface determined as F (x, y, z) = k for k = constant.


Then F is everywhere normal or orthogonal to S. In particular, if P = (x0 , y0 , z0 )
S, then the equation of the tangent plane to S at p is:
Fx (x0 , y0 , z0 )(x x0 ) + Fy (x0 , y0 , z0 )(y y0 ) + Fz (x0 , y0 , z0 )(z z0 ) = 0

(1)

Example 65 Find the equations of the tangent plane and normal line at the point
(2, 1, 3) to the ellipsoid
x2
z2
+ y2 +
= 3.
4
9
Solution : The ellipsoid is the level surface (with k = 3) of the function
F (x, y, z) =

x2
z2
+ y2 + .
4
9

Therefore, we have
Fx (x, y, z) =

x
2

Fy (x, y, z) = 2y

Fz (x, y, z) =

2z
9

2
Fz (2, 1, 3) = .
3
Then Equation 1 in Theorem 64 gives the equation of the tangent plane at
(2, 2, 3) as
2
1(x + 2) + 2(y 1) (z + 3) = 0,
3
which simplifies to 3x 6y + 2z = 18 = 0.
Since F (2, 1, 3) = h1, 2, 32 i, the vector equation of the normal line is:
Fx (2, 1, 3) = 1

Fy (2, 1, 3) = 2

2
L(t) = h2, 1, 3i + th1, 2, i.
3
Definition 66 A function of two variables has a local maximum at (a, b) if
f (x, y) f (a, b) when (x, y) is near (a, b). (This means that f (x, y) f (a, b)
for all points (x, y) in some disk with center (a, b).) The number f (a, b) is called
a local maximum value. If f (x, y) f (a, b) for all f (x, y) in the domain of f ,
then f has an absolute maximum at (a, b). If f (x, y) f (a, b) when (x, y) is near
(a, b), then f (a, b) is a local minimum value. If f (x, y) f (a, b) for all (x, y) in
the domain of f , then f has an absolute minimum at (a, b).

SOME PRACTICE PROBLEMS SOLVED.

16

The next theorem explains how to find local maxima and local minima for a
function in two variables.
Theorem 67 If f has a local maximum of minimum at (a, b) and the first-order
partial derivatives of f exist there, then fx (a, b) = 0 and fy (a, b) = 0.
Definition 68 A point (a, b) is called a critical point of f (x, y) if fx (a, b) =
fy (a, b) = 0.
The next theorem gives a method for testing critical points of a function f (x, y)
to see if they represent local minima, local maxima or saddle points (a critical point
(a, b) is a saddle point if the Hessian D defined in the next theorem is negative).
Theorem 69 (Second Derivative Test) Suppose the second partial derivatives
of f are continuous on a disk with center (a, b), and suppose that fx (a, b) = 0 and
fy (a, b) = 0 (that is, (a, b) is a critical point of f ). Let
D = D(a, b) = fxx (a, b)fyy (a, b) [fxy(a,b) ]2 .
(a) If D > 0 and fxx (a, b) > 0, then f (a, b) is a local minimum.
(b) If D > 0 and fxx (a, b) < 0, then f (a, b) is a local maximum.
(c) If D < 0, then f (a, b) is a saddle point.
To remember the formula for D its

f
f
D = xx xy
fyx fyy

helpful to write it as a determinant:




= fxx fyy (fxy )2 .

Example 70 Find the local maximum and minimum values and saddle points of
f (x, y) = x4 + y 4 4xy + 1.
Solution : We first locate the critical points:
fx = 4x3 4y

fy = 4y 3 4x.

Setting these partial derivatives equal to 0, we obtain the equations


x3 y = 0

y 3 x = 0.

To solve these equations we substitute y = x3 from the first equation into the second
one. This gives
0 = x9 x = x(x8 1) = x(x4 1)(x4 + 1) = x(x2 1)(x2 + 1)(x4 + 1),
so there are three real roots: x = 0, 1, 1. The three critical points are (0, 0), (1, 1),
and (1, 1).
Next we calculate the second partial derivatives and D(x, y):
fxx = 12x2

fxy = 4

fyy = 12y 2

D(x, y) = fxx fyy (fxy )2 = 144x2 y 2 16.


Since D(0, 0) = 16 < 0, it follows from case (c) of the Second Derivative Test
that the origin is a saddle point; hence, f has no local maximum or minimum at
(0, 0). Since D(1, 1) = 128 > 0 and fxx (1, 1) = 12 > 0, we see from case (a) of the
test that f (1, 1) = 1 is a local minimum. Similarly, we have D(1, 1) = 128 > 0
and fxx (1, 1) = 12 > 0, so f (1, 1) = 1 is also a local minimum value.

SOME PRACTICE PROBLEMS SOLVED.

17

Definition 71 A subset D R2 is closed if it contains all of its boundary points.


Definition 72 A subset D R2 is bounded if it is contained within some disk
in the plane.
Theorem 73 (Extreme Value Theorem for Functions of Two Variables) If
f is continuous on a closed, bounded set D in R2 , then f attains an absolute maximum value f (x1 , y1 ) and an absolute minimum value f (x2 , y2 ) at some points (x1 , y1 )
and (x2 , y2 ) in D.
To find the absolute maximum and minimum values of a continuous function f
on a closed, bounded set D:
1. Find the values of f at the critical points of f in D.
2. Find the extreme values of f on the boundary of D.
3. The largest of the values from steps 1 and 2 is the absolute maximum value;
the smallest of these values is the absolute minimum value.
The next theorem is stated for a function f of three variables but there is a
similar theorem for a function of two variables (see Example 75 below).
Theorem 74 (Method of Lagrange Multipliers) To find the maximum and minimum values of f (x, y, z) subject to the constraint g(x, y, z) = k (assuming that these
extreme values exist and g 6= 0 on the surface g(x, y, z) = k):
1. Find all values of x, y, z, and such that
f (x, y, z) = g(x, y, z)
and
g(x, y, z) = k.
2. Evaluate f at all the points (x, y, z) that result from step 1. The largest of
these values is the maximum value of f ; the smallest is the minimum value of
f.
Example 75 Find the extreme values of the function f (x, y) = x2 + 2y 2 on the
circle x2 + y 2 = 1.
Solution : We are asked for the extreme values of f subject to the constraint
g(x, y) = x2 +y 2 = 1. Using Lagrange multipliers, we solve the equations f = g,
g(x, y) = 1, which can be written as
fx = gx

fy = gy

g(x, y) = 1

or as
2x = 2x

(2)

4y = 2y

(3)

x2 + y 2 = 1.

(4)

From (2) we have x = 0 or = 1. If x = 0, then (4) gives y = 1. If = 1, then


y = 0 from (3), so then (4) gives x = 1. Therefore, f has possible extreme values

SOME PRACTICE PROBLEMS SOLVED.

18

at the points (0, 1), (0, 1) (1, 0), and (1, 0). Evaluating f at these four points,
we find that
f (0, 1) = 2

f (0, 1) = 2

f (1, 0) = 1

f (1, 0) = 1.

Therefore, the maximum value of f on the circle x2 + y 2 = 1 is f (0, 1) = 2 and


the minimum value is f (1, 0) = 1.
Example 76 Find the extreme values of f (x, y) = x2 + 2y 2 on the disk x2 + y 2 1.
Solution : We will compare the values of f at the critical points with values at the
points on the boundary. Since fx = 2x and fy = 4y, the only critical point is (0, 0).
We compare the value of f at that point with the extreme values on the boundary
from Example 75:
f (0, 0) = 0

f (1, 0) = 1

f (0, 1) = 2.

Therefore, the maximum value of f on the disk x2 + y 2 1 is f (0, 1) = 2 and the


minimum value is f (0, 0) = 0.
We now start the second material for midterm 2 which concerns double integrals.
For a positive, continuous function f (x, y) defined on a closed and bounded domain
D R2 , we denote by
Z Z
f (x, y) dA,
D

the volume under the graph of f (x, y) over D. This volume for a rectangle R =
{(x, y) | a x b, c y d} = [a, b] [b, c] R2 can be estimated by the
following Midpoint Rule for Double Integrals described in the next theorem. We
also use this rule for defining the double integral when f (x, y) is not necessarily
positive.
Theorem 77 (Midpoint Rule for Double Integrals) Let m, n be positive integers. Let x0 = a < x1 < x2 < . . . < xm = b be a division of [a, b] into n intervals
[xi , xi + 1] of equal width x = ba
m . Similarly, let y0 = c < y1 < y2 < . . . < yn = d
be a division of [c, d] into m intervals [yj , yj+1 ] of equal widths y = dc
n . Then:
Z Z
m X
n
X
f (x, y) dA
f (xi , yj ) A,
R

i=1 j=1

where xi is the midpoint of [xi1 , xi ] and yj is the midpoint of [yj1 , yj ]. Furthermore, the right-hand side above converges to the left-hand side as m, n
Definition 78 If f is a continuous function of two variables, then its average value
on a domain D R2 is:
RR
y) dA
D f (x, R
R
.
Area(D) =
D dA
Definition 79 The iterated integral of f (x, y) on a rectangle R = [a, b] [c, d] is
Z bZ d
Z dZ b
f (x, y) dy dx or
f (x, y) dx dy.
a

RbRd

Rd
One calculates the integral a c f (x, y) dy dx by first calculating A(x) = c f (x, y) dy,
Rb
holding x constant, and then calculating a A(x) dx and similarly, for calculating the
other integral.

SOME PRACTICE PROBLEMS SOLVED.

19

Example 80 Evaluate the iterated integral.


3Z 2

Z
0

x2 y dy dx

Solution : Regarding x as a constant, we obtain


 2


 2
Z 2
2 y=2
3
2
2y
2 1
2 2
x y dy = x
x
= x2 .
=x
2
2
2
2
1
y=1
Thus, the function A in the preceding discussion is given by A(x) = 32 x2 in this
example. We now integrate this function of x from 0 to 3:

3
Z 3
Z 3 Z 2
Z 3Z 2
3 2
x3
27
2
2
x y dy dx =
x y dy dx =
x dx =
= .
2 0
2
0 2
0
1
0
1
Example 81 Evaluate the iterated integral.
Z 2Z 3
x2 y dx dy.
1

Solution : Here we first integrate with respect to x:


2

Z 2
Z 2Z 3
Z 2 Z 3
Z 2  3 x=3
y2
27
x
2
2
dy =
9y dy = 9
= .
y
x y dx dy =
x y dx dy =
3 x=0
2 1
2
1
1
0
1
0
1
Theorem 82 (Fubinis Theorem) If f is continuous on the rectangle R = {(x, y) |
a x b, c y d}, then
Z Z
Z bZ d
Z dZ b
f (x, y) dA =
f (x, y) dy dx =
f (x, y) dx dy.
R

More generally, this is true if we assume that f is bounded on R, f is discontinuous


only on a finite number of smooth curves, and the iterated integrals exist.
RR
2
Example 83 Evaluate the double integral
R (x 3y ) dA, where R = {(x, y) |
0 x 2, 1 y 2}.
Solution : Fubinis Theorem gives
Z Z
Z 2Z 2
Z
2
2
(x 3y ) dA =
(x 3y ) dy dx =
R

Z
=
0

2

xy y 3

y=2
y=1

dx

2
x2
(x 7) dx =
7x = 12.
2
0

Example 84 Find the volume of the solid S that is bounded by the elliptic paraboloid
x2 + 2y 2 + z = 16, the planes x = 2 and y = 2, and the three coordinate planes.
Solution : We first observe that S is the solid that lies under the surface z =
16 x2 y 2 and above the square R = [0, 2] [0, 2]. We are now in a position to
evaluate the double integral using Fubinis Theorem. Therefore,
Z Z
Z 2Z 2
2
2
V =
(16 x 2y ) dA =
(16 x2 2y 2 ) dx dy
R

SOME PRACTICE PROBLEMS SOLVED.


Z

20

1
[16x x3 2y 2 x]x=2
x=0 dy
3
0


Z 2
88
88 4 3 2
( y 4y 2 ) dy =
=
y
= 48.
3
3
3
0
0
=

In general, for any Rcontinuous


function f (x, y) on a closed and bounded domain
R
2
D R , the integral D f (x, y) dA is defined and it is equal to the area under
the graph of f (x, y) on D when the function is positive. There are two cases for D,
called type I and type II, where the integral
Z Z
f (x, y) dA
D

can be calculated in a straightforward manner.


Definition 85 A plane region D is said to be of type I, if it can be expressed as
D = {(x, y) | a x b, g1 (x) y g2 (x)},
where g1 (x) and g2 (x) are continuous.
Definition 86 A plane region D is said to be of type II, if it can be expressed as
D = {(x, y) | c y d, h1 (y) x h2 (y)},
where h1 and h2 are continuous.
Theorem 87 If f is continuous on a type I region D such that
D = {(x, y) | a x b, g1 (x) y g2 (x)},
then

Z bZ

Z Z

g2 (x)

f (x, y) dA =

f (x, y) dy dx.

Theorem 88
Z Z

g1 (x)

d Z h2 (y)

f (x, y) dA =
D

f (x, y) dx dy.
c

h1 (y)

where D is a type II region given by Definition 86.


RR
Example 89 Evaluate
D (x + 2y) dA, where D is the region bounded by the
parabolas y = 2x2 and y = 1 + x2 .
Solution : The parabolas intersect when 2x2 = 1 + x2 , that is x2 = 1, so x = 1.
We note that the region D, is a type I region but not a type II region and we can
write
D = {(x, y) 1 x 1, 2x2 y 1 + x2 }.
Since the lower boundary is y = 2x2 and the upper boundary is y = 1+x2 , Definition
85 gives
Z Z
Z 1 Z 1+x2
Z 1

y=1+x2
(x + 2y) dA =
(x + 2y) dy dx =
xy + y 2 y=2x2 dx
1

2x2

(3x4 x3 + 2x2 + x + 1) dx

1

x5 x4
x3 x2
32
= 3

+2 +
+ x = .
5
4
3
2
15
1

SOME PRACTICE PROBLEMS SOLVED.

21

Example 90 Find the volume of the solid that lies under the paraboloid z = x2 +y 2
and above the region D in the xy-plane bounded by the line y = 2x and the parabola
y = x2 .
Solution 1: We see that D is a type I region and
D = {(x, y) | 0 x 2, x2 y 2x}.
Therefore, the volume under z = x2 + y 2 and above D is
Z Z
Z 2 Z 2x
2
2
V =
(x + y ) dA =
(x2 + y 2 ) dy dx
D

x2

y=2x

Z 2
2
y3
(2x)3
(x2 )3
2
2
2 2
x y+
=
x (2x) +
dx
dx =
x x
3 y=x2
3
3
0
0

2
Z 2 6
x
x7 x5 7x4
14x3
216
4
x +
dx =
=
+
=
.
3
3
21
5
6 0
35
0
Solution 2: We see that D can also be written as a type II region:
1

D = {(x, y) | 0 y 4, y x y}.
2
Therefore, another expression for V is
Z Z
Z 4 Z y
2
2
V =
(x + y ) dA =
(x2 + y 2 ) dx dy
Z

x=y

4 3
x

1
y
2
3

5
y2
y3
y3
=
+ y2x
dy =
+ y2

3
3
24
2
0
0
x= 21 y
4
2 5 2 7 13 4
216
=
y2 + y2 y
=
.
15
7
96
35
0
R1R1
Example 91 Evaluate the iterated integral 0 x sin(y 2 ) dy dx.

!
dy

Solution : If we try toR evaluate the integral as it stands, we are faced with the
2
task of
R first 2evaluating sin(y ) dy. But its impossible to do so in finite terms
since sin(y ) dy is not an elementary function. So we must change the order of
integration. This is accomplished by first expressing the given iterated integral as a
double integral. We have
Z 1Z 1
Z Z
2
sin(y ) dy dx =
sin(y 2 ) dA,
0

where
D = {(x, 0 | 0 x 1, x y 1)}.
We see that an alternative description of D is
D = {(x, y) | 0 y 1, 0 x y}.
This enables us to express the double integral as an iterated integral in the reverse
order:
Z 1Z 1
Z Z
Z 1Z y
Z 1

x=y
2
2
2
sin(y ) dy dx =
sin(y ) dA =
sin(y ) dx dy =
x sin(y 2 ) x=0 dy
0

Z
=
0

1
y sin(y 2 ) dy = cos(y 2 )
2

1
0

1
= (1 cos 1).
2

BASIC MATERIAL.

22

Review guide for the final exam in Math 233


3

Basic material.

This review includes the remainder of the material for math 233. The final exam
will be a cumulative exam with many of the problems coming from the material
covered beginning approximately with chapter 15.4 of the book.
We first recall polar coordinates formulas given in chapter 10.3. The coordinates
of a point (x, y) R3 can be described by the equations:
x = r cos()
y = r sin(),
(5)
p
where r = x2 + y 2 is the distance from the origin and ( xr , yr ) is (cos(), sin()) on
the unit circle. Note that r 0 and can be taken to lie in the interval [0, 2).
To find r and when x and y are known, we use the equations:
r 2 = x2 + y 2

tan() =

y
.
x

(6)

Example 92 Convert the point (2, 3 ) from polar to Cartesian coordinates.


Solution : Since r = 2 and = 3 , Equations 5 give

1
=2 =1
3
2

3
y = r sin() = 2 sin = 2
= 3.
3
2

Therefore, the point is (1, 3) in Cartesian coordinates.


x = r cos() = 2 cos

Example 93 Represent the point with Cartesian coordinates (1, 1) in terms of


polar coordinates.
Solution : If we choose r to be positive, then Equations 6 give
p
p

r = x2 + y 2 = 12 + (1)2 = 2
tan() =

y
= 1.
x

Since the point (1, 1) lies in the fourth quadrant, we can choose = 4 or =

Thus, one possible answer is ( 2, 4 ); another is (r, ) = ( 2, 7


4 ).

7
4 .

The next theorem describes how to calculate the integral of a function f (x, y)
over a polar rectangle. Note that dA = r dr d.
Theorem 94 (Change to Polar Coordinates in a Double Integral) If f is continuous on a polar rectangle R given by 0 a r b, , where
0 2, then
Z Z
Z Z b
f (x, y)dA =
f (r cos(), r sin())r dr d.
R

RR
2
Example 95 Evaluate
R (3x + 4y )dA, where R is the region in the upper half2
2
plane bounded by the circles x = y = 1 and x2 + y 2 = 4.

BASIC MATERIAL.

23

Solution : The region R can be described as


R = {(x, y) | y 0, 1 x2 + y 2 4}.
It is a half-ring and in polar coordinates it is given by 1 r 2, 0 .
Therefore, by Theorem 94,
Z Z
Z Z 2
2
(3x + 4y )dA =
(3r cos() + 4r2 sin2 ())r dr d
R

(3r2 cos() + 4r3 sin2 ()) dr d

=
0

Z
=

[r cos() + r sin

()]r=2
r=1

(7 cos() + 15 sin2 ()) d

d =
0

15
(1 cos(2))] d
2
0

15
15 15

sin(2) =
.
= 7 sin() +
2
4
2
0
[7 cos() +

Example 96 Find the volume of the solid bounded by the plane z = 0 and the
paraboloid z = 1 x2 y 2 .
Solution : If we put z = 0 in the equation of the paraboloid, we get x2 + y 2 = 1.
This means that the plane intersects the paraboloid in the circle x2 + y 2 = 1, so the
solid lies under the paraboloid and above the circular disk D given by x2 + y 2 1.
In polar coordinates D is given by 0 r 1, 0 2. Since 1 x2 y 2 = 1 r2 ,
the volume is
Z Z
Z 2 Z 1
2
2
V =
(1 x y )dA =
(1 r2 )r dr d
D

=
0

r2 r4
(r r ) dr d = 2

2
4


1
=
0

.
2

The next theorem extends our previous application of Fubinis theorem for type
II regions.
Theorem 97 If f continuous on a polar region of the form
D = {(r, ) | , h1 () r h2 ()}
then
Z Z

h2 ()

f (x, y)dA =
D

f (r cos(), r sin())r dr d

h1 ()

The next definition describes the notion of a vector field. We have already seen
an example of a vector field associated to a function f (x, y) defined on a domain
D R2 , namely the gradient vector field f (x, y) = hfx (x, y), fy (x, y)i. In nature
and in physics, we have the familiar examples of the velocity vector field in weather
and force vector fields that arise in gravitational fields, electric and magnetic fields.

BASIC MATERIAL.

24

Definition 98 Let D be a set in R2 (a plane region). A vector field on R2 is a


function F that assigns to each point (x, y) in D a two-dimensional vector F(x, y).
Definition 99 Let E be a subset of R3 . A vector field on R3 is a function F that
assigns to each point (x, y, z) in E a three-dimensional vector F(x, y, z).
Note that a vector field F on R3 can be expressed by its component functions.
So if F = (P, Q, R), then:
F(x, y, z) = P (x, y, z)i + Q(x, y, z)j + R(x, y, z)k.
We now describe our first kind of line integral. These type of integrals arise
form integrating a function along a curve C in the plane or in R3 . The type of line
integral described in the next definition is called a line integral with respect to
arc length.
Definition 100 Let C be a smooth curve in R2 . Given n, consider n equal subdivisions of lengths si ; let (xi , yi ) denote the midpoints of the i-th subdivision. If f
is a real valued function defined on C, then the line integral of f along C is
Z
f (x, y) ds = lim

n
X

f (xi , yi )si ,

j=1

if this limit exists.


The following formula can be used to evaluate this type of line integral.
Theorem 101 Suppose f (x, y) is a continuous function on a differentiable curve
C(t), C : [a, b] R2 . Then
s 
 2
Z b
Z
dx 2
dy
f (x, y) ds =
f (x(t), y(t))
+
dt
dt
dt
a
C
In the above formula,
s

dx
dt

2


+

dy
dt

2
,

is the speed of C(t) at time t.


R
Example 102 Evaluate C 2x ds, where C consists of the arc C1 of the parabola
y = x2 from (0, 0) to (1, 1).
Solution : We can choose x as the time parameter and the equations for C become
y = x2

x=x

0x1

Therefore,
s


dy 2
2x ds =
2x
+
dx
dx
C1
0

1
Z 1 p
1 2
5 51
2 32
2
=
2x 1 + 4x dx = (1 + 4x )
=
.
4 3
6
0
0
Z

dx
dx

2

BASIC MATERIAL.

25

Actually for what we will studying next, another type of line integral will be
important. These line integrals are called line integrals of f along C with
respect to x and y. They are defined respectively for x and y by the following
limits:
Z
n
X
f (x, y) dx = lim
f (xi , yi )xi
n

Z
f (x, y) dy = lim

i=1
n
X

f (xi , yi )yi .

i=1

The following formulas show how to calculate these new type line integrals. Note
that these integrals depend on the orientation of the curve C, i.e., the initial and
terminal points.
Theorem 103
Z

Z
f (x, y) dx =

Z
f (x, y) dy =

Example 104 Evaluate


(5, 3) to (0, 2)

f (x(t), y(t))x0 (t) dt

a
b

f (x(t), y(t))y 0 (t) dt.

R
C

y 2 dx + x dy, where C = C1 is the line segment from

Solution : A parametric representation for the line segment is


x = 5t 5,

y = 5t 3,

0t1

Then dx = 5 dt, dy = 5 dt, and Theorem 103 gives


Z
Z 1
2
y dx + x dy =
(5t 3)2 (5dt) + (5t 5)(5 dt)
C1

=5

(25t2 25t + 4) dt

1
25t3 25t2
5
=5

+ 4t = .
3
2
6
0
R
Example 105 Evaluate C y 2 dx + x dy, where C = C2 is the arc of the parabola
x = 4 y 2 from (5, 3) to (0, 2).


Solution : Since the parabola is given as a function of y, lets take y as the parameter and write C2 as
x = 4 y2

y = y,

3 y 2.

Then dx = 2y dy and by Theorem 103 we have


Z
Z 2
2
y dx + x dy =
y 2 (2y) dy + (4 y 2 ) dy
3

C2

=
3

(2y 3 y 2 + 4) dy

BASIC MATERIAL.

26
2

y4 y3
+ 4y
=
2
3


5
= 40 .
6

One can also define in a similar manner the line integral with respect to arc
length of a function f along a curve C in R3 .
Theorem 106
s 
 2  2
Z b
Z
dx 2
dy
dz
f (x(t), y(t), z(t))
f (x, y, z) ds =
+
+
dt
dt
dt
dt
a
C
Z
=
P (x, y, z) dx + Q(x, y, z) dy + R(x, y, z) dz,
C

where f (x, y, z) = hP (x, y, z), Q(x, y, z), R(x, y, z)i.


The next example demonstrates how to calculate a line integral of a
function with respect to x, y and z.
R
Example 107 Evaluate c y dx + z dy + x dz,, where C consists of the line segment
C1 from (2, 0, 0) to (3, 4, 5) followed by the vertical line segment C2 from (3, 4, 5) to
(3, 4, 0).
Solution : We write C1 as
r(t) = (1 t)h2, 0, 0i + th3, 4, 5i = h2 + t, 4t, 5ti
or, in parametric form, as
x=2+t

y = 4t

Thus
Z

Z
y dx + z dy + x dz =

C1

z = 5t

0 t 1.

(4t) dt + (5t)4 dt + (2 + t)5 dt


0

Z
=
0

t2
(10 + 29t) dt = 10t + 29
2

1
= 24.5.
0

Likewise, C2 can be written in the form


r(t) = (1 t)h3, 4, 5i + th3, 4, 0i = h3, 4, 5 5ti
or
x = 3 y = 4 z = 5 5t dz = 5 dt,
Then dx = 0 = dy, so
Z

2(5) dt = 15.

y dx + z dy + x dz =
C2

0 t 1.

Adding the values of these integrals, we obtain


Z
y dx + z dy + x dz = 24.5 15 = 9.5.
C=C1 C2

We now get to our final type of line integral which can be considered to be a
line integral of a vector field. This type of integral is used to calculate the work
W done by a force field F in moving a particle along a smooth curve C.

BASIC MATERIAL.

27

Theorem 108 If C is given by the vector equation r(t) = x(t)i + y(t)j + z(t)k on
the interval [a, b], then the work W can be calculated by
b

Z
W =

F(r(t)) r0 (t) dt,

where is the dot product.


In general, we make the following definition which is related to the formula in
the above theorem.
Definition 109 Let F be a continuous vector field defined on a smooth curve C
given by a vector function r(t), a t b. Then the line integral of F along C is
b

F T ds;

F(r(t)) r (t) dt =

F dr =

here, T is the unit tangent vector field to the parameterized curve C.


Example 110 Find the work done by the force field F(x, y) = x2 i xyj in moving
a particle along the quarter-circle r(t) = cos t i + sin t j, 0 t 2 .
Solution : Since x = cos t and y = sin t, we have
F(r(t)) = cos2 ti cos t sin tj
and
r0 (t) = sin ti + cos tj.
Therefore, the work done is
Z

Z
F dr =

F(r(t)) r (t)dt =

Example 111 Evaluate


twisted cubic given by

(2 cos2 t sin t)dt

=2
R

cos3 t
3

 2
0

2
= .
3

F dr, where F(x, y, z) = xyi + yzj + zxk and C is the


y = t2

x=t

z = t3

0 t 1.

Solution : We have
r(t) = ti + t2 j + t3 k
r0 (t) = i + 2tj + 3t2 k
F(r(t)) = t3 i + t5 j + t4 k.
Thus,
Z

Z
F dr =

Z
=
0

F(r(t)) r0 (t)dt

(t3 + 5t6 )dt =

t4 5t7
+
4
7

1
=
0

27
.
28

BASIC MATERIAL.

28

Theorem 112 If C in R3 is parameterized by r(t) and F = P i + Qj + Rk, then


Z
Z
P dx + Qdy + Rdz.
F dr =
C

We now apply the material covered so far on line integrals to obtain several
versions of the fundamental theorem of calculus in the multivariable setting. Recall
that the fundamental theorem calculus can be written as
Z b
F0 (x)dx = F (b) F (a),
a

when F0 (x) is continuous on [a, b].


Theorem 113 Let C be a smooth curve given by the vector function r(t), a t
b. Let f be a differentiable function of two or three variables whose gradient vector
f is continuous on C. Then
Z
f dr = f (r(b)) f (r(a)).
C

For further discussion, we make the following definitions.


Definition 114 A curve r : [a, b] R3 (or R3 ) closed if r(a) = r(b).
Definition 115 A domain D R3 (or R2 ) is open if for any point p in D, a small
ball (or disk) centered at p in R3 (in R2 ) is contained in D.
Definition 116 A domain D R3 (or R2 ) is connected if any two points in D
can be joined by a path contained inside D.
Definition 117 A curve r : [a, b] R3 (or R2 ) is a simple curve if it doesnt
intersect itself anywhere between its end points (r(t1 ) 6= r(t2 ) when a < t1 < t2 < b).
Definition 118 An open, connected region D R2 is a simply-connected region
if any simple closed curve in D encloses only points that are in D.
Definition 119 A vector field F is called a conservative vector field if it is the
gradient of some scalar function f (x, y); the function f (x, y) is called a potential
function for F. For example, for f (x, y) = xy + y 2 , f = hy, x + 2yi and so,
F(x, y) = yi + (x + 2y)j is a conservative vector field.
Definition 120
If F is a continuous vector field Rwith domainR D, we say that the
R
line integral C F dr is independent of path if C1 F dr = C2 F dr for any two
paths C1 and C2 in D with the same initial and the same terminal points.
We now states several theorems that you should know for the final exam.
R
R
Theorem 121 C F dr is independent of path in D if and only if C F dr = 0 for
every closed path C in D.
Theorem 122
R Suppose F is a vector field that is continuous on an open connected
region D. If C F dr is independent of path in D, then F is a conservative vector
field on D; that is, there exists a function f such that f = F.

BASIC MATERIAL.

29

Theorem 123 If F(x, y) = P (x, y)i+Q(x, y)j is a conservative vector field, where P
and Q have continuous first-order partial derivatives on a domain D, then throughout
D we have
P
Q
=
.
y
x
Theorem 124 Let F = P i + Qj be a vector field on an open simply-connected
region D. Suppose that P and Q have continuous first-order derivatives and
Q
P
=
y
x

throughout D.

Then F is conservative.
Example 125 Determine whether or not the vector field F(x, y) = (xy)i+(x2)j
is conservative.
Solution : Let P (x, y) = x y and Q(x, y) = x 2. Then
P
= 1
y
Since

P
y

6=

Q
x ,

Q
= 1.
x

F is not conservative by Theorem 123.

Example 126 Determine whether or not the vector field F(x, y) = (3 + 2xy)i +
(x2 3y 2 )j is conservative.
Solution : Let P (x, y) = 3 + 2xy and Q(x, y) = x2 3y 2 . Then
P
Q
= 2x =
.
y
x
Also, the domain of F is the entire plane (D = R2 ), which is open and simplyconnected. Therefore, we can apply Theorem 124 and conclude that F is conservative.
Attention! You will likely have a problem on the final exam which is similar to
the one described in the next example.
Example 127
F = f .

(a) If F(x, y) = (3 + 2xy)i + (x2 3y 2 )j, find a function f such that

R
(b) Evaluate the line integral C F dr, where C is the curve given by
r(t) = et sin t i + et cos t j, 0 t .
Solution :
(a) From Example 126 we know that F is conservative and so there exists a function
f with f = F, that is,
fx (x, y) = 3 + 2xy
(7)
fy (x, y) = x2 3y 2

(8)

Integrating (7) with respect to x, we obtain


f (x, y) = 3x + x2 y + g(y).

(9)

BASIC MATERIAL.

30

Notice that the constant of integration is a constant with respect to x, that is,
it is a function of y, which we have called g(y).
Next we differentiate both sides of (9) with respect to y:
fy (x, y) = x2 + g 0 (y).

(10)

Comparing (8) and (10), we see that


g 0 (y) = 3y 2 .
Integrating with respect to y, we have
g(y) = y 3 + K
where K is a constant. Putting this in (9), we have
f (x, y) = 3x + x2 y y 3 + K
as the desired potential function.
(b) To apply Theorem 113 all we have to know are the initial and terminal points
of C, namely, r(0) = (0, 1) and r() = (0, e ). In the expression for f (x, y)
in part (a), any value of the constant K will do, so lets choose K = 0. Then
we have
Z
Z
F dr =
f dr = f (0, e ) f (0, 1)
C

= e3 (1) = e3 + 1.
This method is much shorter than the straightforward method for evaluating
line integrals described in Theorem 103.
Definition 128 A simple closed parameterized curve C in R2 always bounds a
bounded simply-connected domain D. We say that C is positively oriented if for
the parametrization r(t) of C, the region D is always on the left as r(t) traverses C.
Note that this parametrization is the counterclockwise one on the boundary of unit
disk D = {(x, y) | x2 + y 2 1}.
The next theorem is a version of the fundamental theorem of calculus, since it
allows one to carry out a two-dimensional integral on a domain D by calculating a
related integral one-dimensional on the boundary of D. There will be at least one
final exam problem related to the following theorem.
Theorem 129 (Greens Theorem) Let C be a positively oriented, piecewisesmooth, simple closed curve in the plane and let D be the region bounded by C.
If P and Q have continuous partial derivatives on an open region that contains D,
then

Z Z 
Z
Q P

dA.
P dx + Qdy =
x
y
C
D
An immediate consequence of Greens Theorem are the area formulas described
in the next theorem.

BASIC MATERIAL.

31

Theorem 130 Let D be a simply-connected domain in the plane with simple closed
oriented boundary curve C. Let A be the area of D. Then:
I
I
I
1
y dx =
x dy =
A=
x dy y dx.
(11)
2 C
C
C
Example 131 Find the area enclosed by the ellipse

x2
a2

y2
b2

= 1.

Solution : The ellipse has parametric equations x = a cos t and y = b sin t, where
0 t 2. Using the third formula in Equation 11, we have
Z
1
x dy y dx
A =
2 C
Z
1 2
=
(a cos t)(b cos t) dt (b sin t)(a sin t) dt
2 0
Z
ab 2
=
dt = ab.
2 0
p
H
Example 132 Use Greens Theorem to evaluate C (3yesin x ) dx+(7x+ y 4 + 1) dy,
where C is the circle
x2 + y 2 = 9.
Solution : The region D bounded by C is the disk x2 + y 2 9, so lets change to
polar coordinates after applying Greens Theorem:
I
p
(3y esin x ) dx + (7x + y 4 + 1) dy
C

Z Z 
p

sin x
4
(7x + y + 1)
(3y e
) dA
=
x
y
D
Z 2 Z 3
=
(7 3) r dr d
0
0
Z 2 Z 3
= 4
d
r dr = 36.
0

PRACTICE PROBLEMS FOR EXAM 1.

32

Practice problems from old exams for math


233
William H. Meeks III
September 1, 2014
Disclaimer: Your instructor covers far more materials that we can possibly fit
into a four/five questions exams. These practice tests are meant to give you an idea
of the kind and varieties of questions that were asked within the time limit of that
particular tests. In addition, the scope, length and format of these old exams might
change from year to year. Users beware! These are NOT templates upon which
future exams are based, so dont expect your exam to contain problems exactly like
the ones presented here. Check the course web page for an update on the material
to be covered on each exam or ask your instructor.

Practice problems for Exam 1.


Fall 2008 Exam
1. (a) Find parametric equations for the line L which contains A(1, 2, 3) and
B(4, 6, 5).
(b) Find parametric equations for the line L of intersection of the planes
x 2y + z = 10 and 2x + y z = 0.
2. (a) Find an equation of the plane which contains the points P (1, 0, 1),
Q(1, 2, 1) and R(2, 0, 1).
(b) Find the distance D from the point (1, 6, 1) to the plane 2x+y2z = 19.
(c) Find the point Q in the plane 2x + y 2z = 19 which is closest to the
point (1, 6, 1). (Hint: You can use part b) of this problem to help find
Q or first find the equation of the line L passing through Q and the point
(1, 6, 1) and then solve for Q.)
3. (a) Find the volume V of the parallelepiped such that the following four
points A = (3, 4, 0), B = (3, 1, 2), C = (4, 5, 3), D = (1, 0, 1) are
vertices and the vertices B, C, D are all adjacent to the vertex A.
(b) Find the center and radius of the sphere x2 4x + y 2 + 4y + z 2 = 8.
4. (a) The position vector of a particle moving in space equals r(t) = t2 i t2 j +
1 2
2 t k at any time t 0.
Find an equation of the tangent line to the curve at the point (4, 4, 2).
(b) Find the length L of the arc traveled from time t = 1 to time t = 4.
(c) Suppose a particle moving in space has velocity
v(t) = hsin t, 2 cos 2t, 3et i
and initial position r(0) = h1, 2, 0i. Find the position vector function r(t).
5. (a) Consider the points A(2, 1, 0), B(3, 0, 2) and C(0, 2, 1). Find the area
of the triangle ABC. (Hint: If you know how to find the area of a
parallelogram spanned by 2 vectors, then you should be able to solve this
problem.

PRACTICE PROBLEMS FOR EXAM 1.

33

(b) Three of the four vertices of a parallelogram are P (0, 1, 1), Q(0, 1, 0)
and R(2, 1, 1). Two of the sides are P Q and P R. Find the coordinates of
the fourth vertex.
Spring 2008 Exam
6. (a) Find an equation of the plane through the points A = (1, 2, 3),
B = (0, 1, 3), and C = (2, 1, 4).
(b) Find the area of the triangle with vertices at points A, B, and C given
above. Hint: the area of this triangle is related to the area of a certain
parallelogram
7. (a) Find the parametric equations of the line passing through the point
(2, 4, 1) that is perpendicular to the plane 3x y + 5z = 77.
(b) Find the intersection point of the line in part (a) and the plane 3x y +
5z = 77.
8. (a) A plane curve is given by the graph of the vector function
u(t) = h1 + cos t, sin ti, 0 t 2.
Find a single equation for the curve in terms of x and y, by eliminating
t.
(b) Consider the space curve given by the graph of the vector function
r(t) = h1 + cos t, sin t, ti, 0 t 2.
Sketch the curve and indicate the direction of increasing t in your graph.
(c) Determine parametric equations for the line T tangent to the graph of
the space curve for r(t) at t = /3, and sketch T in the graph obtained
in part (b).
9. Suppose that r(t) has derivative r0 (t) = h sin 2t, cos 2t, 0i on the interval
0 t 1. Suppose we know that r(0) = h 21 , 0, 1i.
(a) Determine r(t) for all t.
(b) Show that r(t) is orthogonal to r0 (t) for all t.
(c) Find the arclength of the graph of the vector function r(t) on the interval
0 t 1.
10. If r(t) = (2t)i + (t2 6)j ( 13 t3 )k represents the position vector of a moving
object (where t 0 is measured in seconds and distance is measured in feet),
(a) Find the speed and the velocity of the object at time t.
(b) If a second object travels along a path given defined by the graph of the
vector function w(s) = h2, 5, 1i + sh2, 1, 5i, show that the paths of the
two objects intersect at a common point P .
(c) If s = t in part (b), (i.e. the position of the second object is w(t) when
the first object is at position r(t)), do the two objects ever collide?

PRACTICE PROBLEMS FOR EXAM 1.

34

Spring 2007 Exam


11. (a) Find parametric equations for the line which contains A(7, 6, 4) and B(4, 6, 5).
(b) Find the parametric equations for the line of intersection of the planes
x 2y + z = 5 and 2x + y z = 0.
12. (a) Find an equation of the plane which contains the points P (1, 0, 2),
Q(1, 2, 1) and R(2, 0, 1).
(b) Find the distance from the point (1, 0, 1) to the plane 2x + y 2z = 1.
(c) Find the point P in the plane 2x + y 2z = 1 which is closest to the
point (1, 0, 1). (Hint: You can use part b) of this problem to help find
P or first find the equation of the line passing through P and the point
(1, 0, 1) and then solve for P .)
13. (a) Consider the two space curves
r1 (t) = hcos(t 1), t2 1, 2t4 i,

r2 (s) = h1 + ln s, s2 2s + 1, 2s2 i,

where t and s are two independent real parameters. Find the cosine of the
angle between the tangent vectors of the two curves at the intersection
point (1, 0, 2).
(b) Find the center and radius of the sphere x2 + y 2 + 2y + z 2 + 4z = 20.
14. The velocity vector of a particle moving in space equals v(t) = 2ti 2tj + tk
at any time t 0.
(a) At the time t = 4, this particle is at the point (0, 5, 4). Find an equation
of the tangent line to the curve at the time t = 4.
(b) Find the length of the arc traveled from time t = 2 to time t = 4.
(c) Find a vector function which represents the curve of intersection of the
cylinder x2 + y 2 = 1 and the plane x + 2y + z = 4.
15. (a) Consider the points A(2, 1, 0), B(1, 0, 2) and C(0, 2, 1). Find the area
of the triangle ABC. (Hint: If you know how to find the area of a
parallelogram spanned by 2 vectors, then you should be able to solve this
problem.)
(b) Suppose a particle moving in space has velocity
v(t) = hsin t, cos 2t, et i
and initial position r(0) = h1, 2, 0i. Find the position vector function r(t).
Fall 2007 Exam
16. Find the equation of the plane containing the lines
x = 4 4t,

y = 3 t,

z = 1 + 5t

x = 4 t, y = 3 + 2t, z = 1

and

PRACTICE PROBLEMS FOR EXAM 1.

35

17. Find the distance between the point P (3, 2, 7) and the plane given by
4x 6y z = 5.
18. Determine whether the lines L1 and L2 given below are parallel, skew or intersecting. If they intersect, find the point of intersection.
L1 :
L2 :

x
y1
z2
=
=
1
2
3

x3
y2
z1
=
=
4
3
2

19. (a) Suppose a particle moving in space has the velocity


v(t) = h3t2 , 2 sin(2t), et i.
Find the acceleration of the particle. Write down a formula for the speed
of the particle (you do not need to simplify the expression algebraically).
(b) If initially the particle has the position r(0) = h0, 1, 2i, what is the
position at time t?
20. Three of the four vertices of a parallelogram are P (0, 1, 1), Q(0, 1, 0) and
R(3, 1, 1). Two of the sides are P Q and P R. This problem continues on the
next page.
(a) Find the area of the parallelogram.

(b) Find the cosine of the angle between the vector P Q and P R.
(c) Find the coordinates of the fourth vertex.
21. Let C be the parametric curve
x = 2 t2 , y = 2t 1, z = ln t.
This problem continues on the next page.
(a) Determine the point(s) of intersection of C with the xz-plane.
(b) Determine the parametric equation of the tangent line to C at (1, 1, 0).
(c) Set up, but not solve, a formula that will determine the length of C for
1 t 2.
Fall 2006 Exam
22. (a) Find parametric equations for the line which contains A(2, 0, 1) and B(1, 1, 1).
(b) Determine whether the lines l1 : x = 1 + 2t, y = 3t, z = 2 t and
l2 : x = 1 + s, y = 4 + s, z = 1 + 3s are parallel, skew or intersecting.
23. (a) Find an equation of the plane which contains the points P (1, 2, 1),
Q(1, 2, 1) and R(1, 1, 1).
(b) Find the distance from the point (1, 2, 1) to the plane 2x + y 2z = 1.

PRACTICE PROBLEMS FOR EXAM 1.

36

24. (a) Let two space curves


r1 (t) = hcos(t 1), t2 1, t4 i, r2 (s) = h1 + ln s, s2 2s + 1, s2 i,
be given where t and s are two independent real parameters. Find the
cosine of the angle between the tangent vectors of the two curves at the
intersection point (1, 0, 1).
(b) Suppose a particle moving in space has velocity
v(t) = hsin t, cos 2t, et i
and initial position r(0) = h1, 2, 0i. Find the position vector function r(t).
p
2
25. (a) Let f (x, y) = ex y + x 4 y 2 . Find partial derivatives fx , fy and fxy .
(b) Find an equation for the tangent plane of the graph of
f (x, y) = sin(2x + y) + 1
at the point (0, 0, 1).
26. (a) Let g(x, y) = yex . Estimate g(0.1, 1.9) using the linear approximation of
g(x, y) at (x, y) = (0, 2).
(b) Find the center and radius of the sphere x2 + y 2 + z 2 + 6z = 16.
p
(c) Let f (x, y) = 16 x2 y 2 . Draw a contour map of level curves f (x, y) =
k with k = 1, 2, 3. Label the level curves by the corresponding values of
k.
These problems are from older exams
27. Consider the line L through points A = (2, 1, 1) and B = (5, 3, 2). Find
the intersection of the line L and the plane given by 2x 3y + 4z = 13.
28. Two masses travel through space along space curve described by the two vector
functions
r1 (t) = ht, 1 t, 3 + t2 i, r2 (s) = h3 s, s 2, s2 i
where t and s are two independent real parameters.
(a) Show that the two space curves intersect by finding the point of intersection
and the parameter values where this occurs.
(b) Find parametric equation for the tangent line to the space curve r(t) at
the intersection point.
29. Consider the parallelogram with vertices A, B, C, D such that B and C are
adjacent to A. If A = (2, 5, 1), B = (3, 1, 4), D = (5, 2, 3), find the point C.
30. Consider the points A = (2, 1, 0), B = (1, 0, 2) and C = (0, 2, 1).

(a) Find the orthogonal projection proj (AC) of the vector AC onto the

AB

vector AB.
(b) Find the area of triangle ABC.
(c) Find the distance d from the point C to the line L that contains points A
and B.

PRACTICE PROBLEMS FOR EXAM 1.

37

31. Find parametric equations for the line of intersection of the planes x2y+z = 1
and 2x + y + z = 1.
32. Let L1 denote the line through the points (1, 0, 1) and (1, 4, 1) and let L2
denote the line through the points (2, 3, 1) and (4, 4, 3). Do the lines L1
and L2 intersect? If not, are they skew or parallel?
33. (a) Find the volume of the parallelepiped such that the following four points
A = (1, 4, 2), B = (3, 1, 2), C = (4, 3, 3), D = (1, 0, 1) are vertices and
the vertices B, C, D are all adjacent to the vertex A.
(b) Find an equation of the plane through A, B, D.
(c) Find the angle between the plane through A, B, C and the xy plane.
34. The velocity vector of a particle moving in space equals v(t) = 2ti + 2t1/2 j + k
at any time t 0.
(a) At the time t = 0 this particle is at the point (1, 5, 4). Find the position
vector r(t) of the particle at the time t = 4.
(b) Find an equation of the tangent line to the curve at the time t = 4.
(c) Does the particle ever pass through the point P = (80, 41, 13) ?
(d) Find the length of the arc traveled from time t = 1 to time t = 2.
35. Consider the surface x2 + 3y 2 2z 2 = 1.
(a) What are the traces in x = k, y = k, z = k? Sketch a few.
(b) Sketch the surface in the space.
36. Find an equation for the tangent plane to the graph of f (x, y) = y ln x at
(1, 4, 0).
37. Find the distance between the given parallel planes
z = 2x + y 1, 4x 2y + 2z = 3.
38. Identify the surface given by the equation 4x2 + 4y 2 8y z 2 = 0. Draw the
traces and sketch the curve.
39. A projectile is fired from a point 5 m above the ground at an angle of 30
degrees and an initial speed of 100 m/s.
a) Write an equation for the acceleration vector.
b) Write a vector for initial velocity.
c) Write a vector for initial position.
d) At what time does the projectile hit the ground?
e) How far did it travel, horizontally, before it hit the ground?
40. Explain why the limit of f (x, y) = (3x2 y 2 )/(2x4 + y 4 ) does not exist as (x, y)
approaches (0, 0).
41. Find an equation of the plane that passes through the point P (1, 1, 0) and
contains the line given by parametric equations x = 2+3t, y = 1t, z = 2+2t.
42. Find all of the first order and second order partial derivatives of the function.

PRACTICE PROBLEMS FOR EXAM 1.

38

(a) f (x, y) = x3 xy 2 + y
p
(b) f (x, y) = ln(x + x2 + y 2 )
43. Find the linear approximation of the function f (x, y) = xyex at (x, y) = (1, 1),
and use it to estimate f (1.1, 0.9).
44. Find a vector function which represents the curve of intersection of the paraboloid
z = 2x2 + y 2 and the parabolic cylinder y = x2 .
Spring 2009 Exam
45. Given ~a = h3, 6, 2i,

~b = h1, 2, 3i.

a) Write down the vector projection of ~b to ~a. (Hint: Use projections.)


b) Write ~b as a sum of a vector parallel to ~a and a vector orthogonal to ~a.
(Hint: Use projections.)
c) Let be the angle between ~a and ~b. Find cos .
46. Given A = (1, 7, 5), B = (3, 2, 2) and C = (1, 2, 3).
a) Let L be the line which passes through the points A = (1, 7, 5) and
B = (3, 2, 2). Find the parametric equations for L.
(b) A, B and C are three of the four vertices of a parallelogram, while CA and
CB are two of the four edges. Find the fourth vertex.
47. Consider the points P (1, 3, 5), Q(2, 1, 2), R(1, 1, 1) in R3 .
a) Find an equation for the plane containing P , Q and R.
b) Find the area of the triangle with vertex P, Q, R.
48. Find parametric equations for the line of intersection of the planes
x + y + 3z = 1 and x y + 2z = 0.
49. Consider the parameterized curve


r(t) = t, t2 , t3 , t R.
a) Set up an integral for the length of the arc between t = 0 and t = 1. Do
not attempt to evaluate the integral.
b) Write down the parametric equations of tangent line to r(t) at (2, 4, 8).
50. a) Consider the sphere S in R3 given by the equation
x2 + y 2 + z 2 4x 6z 3 = 0.
Find its center C and its radius R. b) What does the equation x2 + z 2 = 4
describe in R3 ? Make a sketch.
51. Jane throws a basketball at an angle of 45o to the horizontal at an initial
speed of 12 m/s, where m denotes meters. It leaves her hand 2 m above the
ground. Assume the acceleration on the ball due to gravity is downward with
magnitude 10 m/s2 and neglect air friction.

PRACTICE PROBLEMS FOR EXAM 1.

39

(a) Find the velocity function v(t) and the position function r(t) of the ball.
Use coordinates in the xy-plane to describe what is happening; assume
Jane is standing with her feet at the point (0, 0) and y represents the
height.
(b) Find the speed of the ball at its highest point.
(c) At what time T does the ball reach its highest point.
Fall 2009 Exam
52. Let v = 2i j + 2k and w = 2i + 6k + 9j.
(a) Find the vector representing the projection of v onto w.
(b) Find cos , where is the angle between v and w.
53. Consider the points P = (0, 3, 3), Q = (1, 3, 2), R = (1, 2, 3).
(a) Find an equation for the plane containing P, Q, R.
(b) Find the area of the triangle with vertices P, Q, R.
54. Let P1 be the plane x + y z = 0 and P2 be the plane x 2y + z = 1.
(a) Find parametric equations for the line of intersection of P1 and P2 .
(b) Find the distance from the origin to the plane P2 .
55. Let r(t) = t2 i + t3 j + t6 k.
(a) Find an equation for the tangent line to the graph at the point given by
t = 1.
(b) Find the unit tangent vector T to the graph at the point given by t = 1.
(c) Write a definite integral that computes the length of the graph of r(t) from
t = 1 to t = 2, but do not attempt to evaluate it.
56. Consider a particle moving with acceleration a(t) = ht, et , sin(t)i.
(a) Find the velocity vector v(t) of the particle, assuming that v(0) = 0.
(b) Find the position vector r(t) of the particle, assuming that r(0) = 0.
Spring 2010 Exam
57. Consider the parallelogram with vertices A, B, C, D such that B and C are
adjacent to A where A = (1, 2, 1), B = (3, 5, 1) and D = (2, 1, 2).
(a) Find the area of the parallelogram.
(b) Find the coordinates of the point C.
58. Consider the points A = (0, 3, 3), B = (1, 3, 2), C = (1, 2, 3).

(a) Find the orthogonal projection proj (AC) of the vector AC onto the

AB

vector AB.
(b) Find the distance d from the point C to the line L that contains points
A and B.

PRACTICE PROBLEMS FOR EXAM 1.

40

59. Let P1 be the plane x + 3y + z = 0 and P2 be the plane 2x + y z = 1.


(a) Find the cosine of the angle between the planes.
(b) Find the parametric equations of the line of intersection between the 2
planes P1 and P2 .
(c) Find the distance from the plane P2 to the origin.
60. Let r(t) = cos(2t)i + sin(2t)j + tk.
(a) What is the length of the curve starting at t = 0 and ending at t = 5.
(b) Find a vector equation for the tangent line to the graph at the point given
by t = 0.
61. Show that the limit lim(x,y)(0,0)

x2 y 2
x2 +y 2

does not exist.

62. Consider the sphere S in R3 given by the equation


x2 + y 2 + z 2 2x 8y 2 = 0
(a) Find the coordinates of its center and its radius.
(b) What does the equation x2 + y 2 = 64 describe in R3 . Make a sketch.
Spring 2012 Exam
63. Given A = (1, 7, 5), B = (3, 0, 2) and C = (1, 2, 3).
(a) Let L be the line which passes through the points A and B. Find the
parametric equations for L.
(b) A, B and C are three of the four vertices of a parallelogram, while AB
and BC are two of the four edges. Find the fourth vertex.
64. Consider the points P (1, 1, 1), Q(2, 1, 2), R(1, 3, 5) in R3 .
(a) Find an equation for the plane containing P , Q and R.
(b) Find the area of the triangle with vertices P, Q, R.
65. Let P1 be the plane x 2y + 2z = 10 and P2 be the plane 2x + y + 2z = 0.
Find the cosine of the angle between the planes.
66. a) Find the distance from the point Q = (1, 6, 1) to the plane 2x+y2z = 19.
(Hint: To do this you can use the vector projection of some vector P~Q, where
P is some point on the plane.)
b) Write the parametric equations of the line L containing the point T (1, 2, 3)
and perpendicular to the plane 2x + y 2z = 19.
c) Find the point of intersection of the line L in part b) with the plane 2x +
y 2z = 19.
67. Find the equation of the sphere with center at the point (1, 2, 3) and which
contains the point (3, 1, 5).
68. Make a sketch of the surface in R3 described by equation y 2 + z 2 = 36. In
your sketch of this surface, include the labeled coordinate axes and the trace
curves on the surface for the planes x = 0 and x = 4.

PRACTICE PROBLEMS FOR EXAM 2.

41

69. Find the equation of the plane which contains the points A(1, 2, 3) and B(1, 0, 4)
and which is also perpendicular to the plane 4x 2y + z = 8.
70. Suppose ~a = h2, 1, 2i and ~b = h8, 2, 0i.
(a) Find the vector projection, call it ~c, of ~b in the direction ~a.
(b) Calculate the vector ~b ~c and then show that it is orthogonal to ~a.

Practice problems for Exam 2.


Fall 2008 Exam
1. (a) For the function f (x, y) = 2x2 + xy 2 , calculate fx , fy , fxy , fxx :

fx (x, y) =
fy (x, y) =
fxy (x, y) =
fxx (x, y) =

(b) What is the gradient f (x, y) of f at the point (1, 2)? f =


(c) Calculate the directional derivative of f at the point (1, 2) in the
direction of the vector v = h3, 4i?
(d) Next evaluate Du f (1, 2) =
(e) What is the linearization L(x, y) of f at (1, 2) ?
(f) Use the linearization L(x, y) in the previous part to estimate f (0.9, 2.1).
2. A hiker is walking on a mountain path. The surface of the mountain is modeled
by z = 100 4x2 5y 2 . The positive x-axis points to East direction and the
positive y-axis points North.
(a) Suppose the hiker is now at the point P (2, 1, 79) heading North, is she
ascending or descending?
(b) When the hiker is at the point Q(1, 0, 96), in which direction on the map
should she initially head to descend most rapidly?
(c) What is her rate of descent when she travels at a speed of 10 meters
per minute in the direction of maximal decent from Q(1, 0, 96) ?
(d) When the hiker is at the point Q(1, 0, 96), in which two directions on
her map can she initially head to neither ascend nor descend (to keep
traveling at the same height)?
Justify your answers.
3. (a) Let f (x, y) be a differentiable function with the following values of the
partial derivatives fx (x, y) and fy (x, y) at certain points (x, y)
x
1
1
1

y
1
2
2

fx (x, y)
2
3
1

fy (x, y)
4
1
3

PRACTICE PROBLEMS FOR EXAM 2.

42

(You are given more values than you will need for this problem.) Suppose
that x and y are functions of variable t: x = t3 ; y = t2 + 1, so that we
may regard f as a function of t. Compute the derivative of f with respect
to t when t = 1.
z
Use the Chain Rule to find v
when u = 1 and v = 1, where
z = x3 y 2 + y 3 x; x = u2 + v 2 , y = u v 2 .
(b) Use the Chain Rule to find

z
v

when u = 1 and v = 1, where

z = x3 y 2 + y 3 x; x = u2 + v 2 , y = u v 2 .
4. Consider the surface x2 + y 2 2z 2 = 0 and the point P (1, 1, 1) which lies on
the surface.
(i) Find the equation of the tangent plane to the surface at P .
(ii) Find the equation of the normal line to the
surface at P .
5. Let
f (x, y) = 2x3 + xy 2 + 6x2 + y 2 .
Find and classify (as local maxima, local minima or saddle points) all critical points of f .
6. A flat circular plate has the shape of the region x2 + y 2 4. The plate
(including the boundary x2 + y 2 = 4) is heated so that the temperature at
any point (x, y) on the plate is given by T (x, y) = x2 + y 2 2x. Find the
temperatures at the hottest and the coldest points on the plate, including the
boundary x2 + y 2 = 4.
Spring 2008 Exam
7. Consider the equation x2 + y 2 /9 + z 2 /4 = 1.
(a) Identify this quadric (i.e. quadratic surface), and graph the portion of the
surface in the region x 0, y 0, and z 0. Your graph should include
tick marks along the three positive coordinate axes, and must clearly show
where the surface intersects any of the three positive coordinate axes.
(b) Calculate zx and zy at an arbitrary point (x, y, z) on the surface.
(c) Determine the equation of the tangent plane to the surface at the point
( 12 , 32 , 1).
8. Given the function f (x, y) = x2 y + yexy .
(a) Find the linearization of f at the point (0, 5) and use it to approximate
the value of f at the point (.1, 4.9). (An unsupported numerical approximation to f (.1, 4.9) will not receive credit.)
(b) Suppose that x(r, ) = r cos and y(r, ) = r sin . Calculate f at r = 5
and = 2 .

PRACTICE PROBLEMS FOR EXAM 2.

43

(c) Suppose a particle travels along a path (x(t), y(t)), and that F (t) =
f (x(t), y(t)) where f (x, y) is the function defined above. Calculate F 0 (3),
assuming that at time t = 3 the particles position is (x(3), y(3)) = (0, 5)
and its velocity is (x0 (3), y 0 (3)) = (3, 2).
p
9. Consider the function f (x, y) = 2 x2 + 4y.
(a) Find the directional derivative of f (x, y) at P = (2, 3) in the direction
starting from P pointing towards Q = (0, 4).
(b) Find all unit vectors u for which the directional derivative
Du f (2, 3) = 0.
(c) Is there a unit vector u for which the directional derivative
Du f (2, 3) = 4? Either find the appropriate u or explain why not.
10. let f (x, y) = 23 x3 + 13 y 3 xy.
(a) Find all critical points of f (x, y).
(b) Classify each critical point as a relative maximum, relative minimum or
saddle; you do not need to calculate the function at these points, but
your answer must be justified by an appropriate calculation.
11. Use the method of Lagrange multipliers to determine all points (x, y) where
the function f (x, y) = 2x2 + 4y 2 + 16 has an extreme value (either a maximum
2
or a minimum) subject to the constraint x4 + y 2 = 4.
Fall 2007 Exam
12. Find the x and y coordinates of all critical points of the function
f (x, y) = 2x3 6x2 + xy 2 + y 2
and use the second derivative test to classify them as local minima, local
maxima or saddle points.
13. A hiker is walking on a mountain path. The surface of the mountain is modeled
by z = 1 4x2 3y 2 . The positive x-axis points to East direction and the
positive y-axis points North. Justify your answers.
(a) Suppose the hiker is now at the point P ( 41 , 21 , 0) heading North, is she
ascending or descending?
(b) When the hiker is at the point Q( 14 , 0, 34 ), in which direction should she
initially head to ascend most rapidly?
14. Find the volume of the solid bounded by the surface z = 6 xy and the planes
x = 2, x = 2, y = 0, y = 3, and z = 0.
15. Let z(x, y) = x2 + y 2 xy where x = s r is a known function of r and s and
y = y(r, s) is an unknown function of r and s. (Note that z can be considered
a function of r and s.) Suppose we know that
y(2, 3) = 3,
Calculate

z
r

y
(2, 3) = 7,
r

when r = 2 and s = 3.

and

y
(2, 3) = 5.
s

PRACTICE PROBLEMS FOR EXAM 2.

44

16. Let F (x, y, z) = x2 2xy y 2 + 8x + 4y z. This problem continues on the


next page.
(a) Write the equation of the tangent plane to the surface given by F (x, y, z) =
0 at the point (2, 1, 5).
(b) Find the point (a, b, c) on the surface at which the tangent plane is horizontal, that is, parallel to the z = 0 plane.
17. Find the points on the ellipse x2 + 4y 2 = 4 that are closest to the point (1, 0).
Fall 2006 Exam
18. (a) Let f (x, y) be a differentiable function with the following values of the
partial derivatives fx (x, y) and fy (x, y) at certain points (x, y):
x
1
1
1

y
1
2
2

fx (x, y)
2
3
1

fy (x, y)
4
1
1

(You are given more values than you will need for this problems.) Suppose
that x and y are functions of variable t:
x = t3 ; y = t2 + 1,
so that we may regard f as a function of t. Compute the derivative of f
with respect to t when t = 1.
(b) Use the Chain Rule to find

z
v

when u = 1 and v = 1, where

z = x3 y 2 + y 3 x; x = u2 + v, y = 2u v.
19. (a) Let f (x, y) = x2 y 3 + y 4 . Find the directional derivative of f at the point
(1, 1) in the direction which forms an angle (counterclockwise) of /6
with the positive x-axis.
(b) Find an equation of the tangent line to the curve x2 y + y 3 5 = 0 at the
point (x, y) = (2, 1).
20. Let
f (x, y) = 2x3 + xy 2 + 5x2 + y 2 .
Find and classify (as local maxima, local minima or saddle points) all critical
points of f .
21. Find the maximum value of f (x, y) = 2x2 + y 2 on the circle x2 + y 2 = 1.
22. Find the volume above the rectangle 1 x 1,
surface z = 5 + x2 + y.
23. Evaluate the integral
Z
0

1Z 1

p
x3 + 1 dx dy

by reversing the order of integration.

2 y 5 and below the

PRACTICE PROBLEMS FOR EXAM 2.

45

These problems from older exams


24. Use Chain Rule to find dz/dt or z/u, z/v.
(1) z = x2 y + 2y 3 , x = 1 + t2 , y = (1 t)2 .
(2) z = x3 + xy 2 + y 3 , x = uv, y = u + v.
25. If z = f (x, y), where f is differentiable, and x = 1 + t2 , y = 3t, compute dz/dt
at t = 2 provided that fx (5, 6) = fy (5, 6) = 1.
26. For the following functions
(1). f (x, y) = x2 y+y 3 y 2 , (2) g(x, y) = x/y+xy, (3) h(x, y) = sin(x2 y)+xy 2 .
(a) Find the gradient.
(b) Find the directional derivative at the point (0, 1) in the direction of v =
h3, 4i.
(c) Find the maximum rate of change at the point (0, 1).
27. Find an equation of the tangent plane to the surface x2 + 2y 2 z 2 = 5 at the
point (2, 1, 1).
28. Find parametric equations for the tangent line to the curve of intersection of
the surfaces z 2 = x2 + y 2 and x2 + 2y 2 + z 2 = 66 at the point (3, 4, 5).
29. Find and classify all critical points (as local maxima, local minima, or saddle
points) of the following functions.
(1) f (x, y) = x2 y 2 + x2 2y 3 + 3y 2 , (2) g(x, y) = x3 + y 2 + 2xy 4x 3y + 5.
30. Find the minimum value of f (x, y) = 3 + xy x 2y on the closed triangular
region with vertices (0, 0), (2, 0) and (0, 3).
31. Use Lagrange multipliers to find the extreme values of the following functions
with the given constraint.
(1) f (x, y) = xy with constraint x2 + 2y 2 = 3;
(2) g(x, y, z) = x + 3y 2z with constraint x2 + 2y 2 + z 2 = 5.
32. Find the following iterated integrals.
R4R2

(1) 1 0 (x + y) dx dy
(2)

R2R1

(3)

R 1 R 2x

(4)

R1R1

(2x + 3y)2 dy dx

x2

(x2 y) dy dx

x3 sin(y 3 ) dy dx (hint: reverse the order of integration)

PRACTICE PROBLEMS FOR EXAM 2.

46

33. Evaluate the following double integrals.


RR
(1)
R cos(x + 2y) dA, R = {(x, y) | 0 x , 0 y /2}
y2

(2)

RR

Re

(3)

RR

Rx

dA, R = {(x, y) | 0 y 1, 0 x y}

p
y 2 x2 dA, R = {(x, y) | 0 y 1, 0 x y}

34. Find the volume.


(1) The solid under the surface z = 4 + x2 y 2 and above the rectangle
R = {(x, y) | 1 x 1, 0 y 2}
(2) The solid under the surface z = 2x + y 2 and above the region bounded by
curves x y 2 = 0 and x y 3 = 0.

Spring 2009 Exam


35. Let f (x, y) = x2 y y 2 2y x2 .
(a) Find all of the critical points of f and classify them as either local maximum, local minimum, or saddle points.
(b) Find the linearization L(x, y) of f at the point (1, 2) and use it to approximate f (0.9, 2.1).
36. Consider the function f (x, y) = x2 2xy + 3y + y 2 .
(a) Find the gradient f (x, y).
(b) Find the directional derivative of f at the point (1, 0) in the direction
h3, 4i.
(c) Compute all second partial derivatives of f .
(d) Suppose x = st2 and y = est . Find

f
s

and

f
t

at s = 2 and t = 1.

37. Consider the function f (x, y) = exy over the region D given by x2 + 4y 2 2.
(a) Find the critical points of f .
(b) Find the extreme values on the boundary of D.
(c) What is the absolute maximum value and absolute minimum value of
f (x, y) on D?
38. (a) Evaluate the following iterated integral.
Z

(x2 y xy) dy dx

(b) Find the volume of the region below z = x2 2xy + 3 and above the
rectangle R = [0, 1] [1, 1].
39. Consider the surface S given by the equation x2 + y 3 + z 2 = 0.

PRACTICE PROBLEMS FOR FINAL EXAM.

47

(a) Give an equation for the tangent plane of S at the point (2, 2, 2).
(b) Give an equation for the normal line to S at the point (2, 2, 2).
Fall 2009 Exam
40. (a) Let f (x, y) = sin(xy)+cos(x+y). Compute an equation for the tangent
plane to the graph of f at the point where x = /4, y = /4.
(b) Let g(x, y, z) = x2 y + y 2 z + xz 2 . Compute the directional derivative at
the point (1, 1, 1) in the direction of the vector 3i + 4k.
2

41. Suppose z = ex +y + sin(x + y 2 ), and x = st, y = s/t. Use the Chain Rule to
find z/t and z/s when s = t = 1.
(a) Use the chain rule to write expressions for z/t and z/s, but do not
evaluate the partial derivatives.
(b) Compute all the partial derivatives you wrote in (a). Your answers may
involve x, y as well as s, t.
(c) Now use the partial derivatives you computed in (b) together with the
formulas in (a) to compute z/t and z/s. Your answer should only
involve the variables s, t.
42. Let f (x, y) = x3 /3 + xy 2 2xy 3x.
(a) Compute the gradient of f .
(b) Find all critical points of f .
(c) For each critical point you found above, classify it as a local maximum,
local minimum, or a saddle point.
43. Find the absolute maximum and minimum values attained by f (x, y) = x2
2x + y 2 4y + 2 on the closed square with vertices (0, 0), (4, 0), (0, 4), (4, 4) (in
other words, the domain {(x, y) | 0 x 4, 0 y 4}).
44. Use the method of Lagrange multipliers to find the maximum and minimum
values attained by the function f (x, y, z) = x + y + z on the ellipsoid 2x2 +
3y 2 + 6z 2 = 1.

Practice problems for Final Exam.


Fall 2008 Exam
1. (a) Consider the points
A = (1, 0, 0), B = (2, 1, 0) and C = (1, 2, 3). Find the parametric
equations for the line L passing through the points A and C.
(b) Find an equation of the plane in R3 which contains the points A, B, C.
(c) Find the area of the triangle T with vertices A, B and C.
2. Find the volume under the graph of f (x, y) = x + 2xy and over the bounded
region in the first quadrant {(x, y) | x 0, y 0} bounded by the curve
y = x2 + 1 and the x and y-axes.

PRACTICE PROBLEMS FOR FINAL EXAM.


3. Let
Z

1Z 2

I=
0

48

sin(y 2 ) dy dx.

2x

(a) Sketch the region of integration.


(b) Write the integral I with the order of integration reversed.
(c) Evaluate the integral I. Show your work.
4. Consider the function F(x, y, z) = x2 + xy 2 + z.
(a) What is the gradient F(x, y, z) of F at the point (1, 2, 1)?
(b) Calculate the directional derivative of F at the point (1, 2, 1) in the
direction of the vector h1, 1, 1i?
(c) What is the maximal rate of change of F at the point (1, 2, 1)?
(d) Find the equation of the tangent plane to the level surface F(x, y, z) = 4
at the point (1, 2, 1).
5. Find the volume V of the solid under the surface z = 1 x2 y 2 and above
the xy-plane.
6. (a) Determine whether the following vector fields are conservative or not.
Find a potential function for those which are indeed conservative.
i. F(x, y) = hx2 + ex + xy, xy sin(y)i.
ii. G(x, y) = h3x2 y + ex + y 2 , x3 + 2xy + 3y 2 i.
7. Evaluate the line integral
Z
yz dx + xz dy + xy dz,
C

where C is the curve starting at (0, 0, 0), traveling along a line segment to
(1, 0, 0) and then traveling along a second line segment to (1, 2, 1).
8. (a) Use Greens Theorem to show that if D R2 is the bounded region with
boundary a positively oriented simple closed curve C, then the area of D
can be calculated by the formula:
Z
1
y dx + x dy
Area =
2 C
(b) Consider the ellipse 4x2 +y 2 = 1. Use the above area formula to calculate
the area of the region D R2 with boundary this ellipse. (Hint: This
ellipse can be parameterized by r(t) = h 21 cos(t), sin(t)i for 0 t 2.)
Spring 2008 Exam
9. Use the space curve ~r(t) = ht2 1, t2 , t/2i for parts (a) and (b)below.
(a) Find the velocity, speed, and acceleration of a particle whose positions
function is ~r(t) at time t = 4.
(b) Find all points where the particle with position vector ~r(t) intersects the
plane x + y 2z = 0.

PRACTICE PROBLEMS FOR FINAL EXAM.

49

10. Let D be the region of the xy plane above the graph of y = x2 and below the
line y = x.
(a) Determine an iterated integral expression for the double integral
Z
xy dA
D

(b) Find an equivalent iterated integral to the one found in part (a) with the
reversed order of integration.
(c) Evaluate one of the two iterated integrals in parts (a,b).
11. Find the absolute maximum and absolute minimum values of
f (x, y) = x2 + 2y 2 2y in the set D = {(x, y) : x2 + y 2 4}.
12. Let D be the region in the first quadrant x, y 0 that lies between the two
circles x2 + y 2 = 4 and x2 + y 2 = 9.
(a) Describe the region D using polar coordinates.
(b) Evaluate the double integral
Z Z
3x + 3y dA.
D
z
z
13. (a) Find x
and y
at the point(1, 0, 1) for the implicit function z determined by the equation

x3 + y 3 + z 3 3xyz = 0.
(b) Is the tangent plane to the surface
x3 + y 3 + z 3 3xyz = 0
at the point (1, 0, 1) perpendicular to the plane 2x + y 3z = 2? Justify
your answer with an appropriate calculation.
14. (a) Consider the vector field G(x, y) = h4x3 + 2xy, x2 i. Show that G is
conservative (i.e. G is a potential or a gradient vector field), and use the
fundamental theorem for line integrals to determine the value of
Z
G dr,
C

where C is the contour consisting of the line starting at (2,-2) and ending
at (1, 1).
(b) Now let T denote the closed contour consisting of the triangle with vertices at (0,0),(1,0), and (1,1) with the counterclockwise orientation, and
let F(x, y) = h 12 y 2 y, xyi. Compute
Z
F dr
C

directly (from the definition of line integral).


(

PRACTICE PROBLEMS FOR FINAL EXAM.

50

(c) Explain how Greens theorem can be used to show that the integral in
(b) must be equal to the area of the region D interior to T .
Fall 2007 Exam
15. Let

4Z 2

Z
I=
0

ex dx dy.

(a) Sketch the region of integration


(b) Write the integral I with the order of integration reversed.
(c) Evaluate the integral I. Show your work.
16. Find the distance from the point (3, 2, 7) to the line
x = 1 + t,

y = 2 t,

z = 1 + 3t.

17. (a) Find the velocity and acceleration of a particle moving along the curve
r(t) = ht, t2 , t3 i
at the point (2, 4, 8).
(b) Find all points where the curve in part (a) intersects the surface z =
3x3 + xy x.
18. Find the volume of the solid which lies below the sphere x2 + y 2 + z 2 = 4,
above the xy-plane, and inside the cylinder x2 + y 2 = 3.
19. Consider the line integral
Z

1 + x dx + 2xy dy,

where C is the triangular path starting from (0, 0), to (2, 0), to (2, 3), and back
to (0, 0). This problem continues on the next page.
(a) Evaluate this line integral directly, without using Greens Theorem.
(b) Evaluate this line integral using Greens theorem.
20. Consider the vector field F = (y 2 /x2 )i (2y/x)j.
(a) Find a function f such that f = F.
(b) Let C be theRcurve given by r(t) = ht3 , sin ti for
line integral C F dr.

t . Evaluate the

Fall 2006 Exam


21. Find parametric equations for the line in which the planes x 2y + z = 1 and
2x + y + z = 1 intersect.
22. Consider the surface x2 + y 2 2z 2 = 0 and the point P (1, 1, 1) which lies on
the surface.

PRACTICE PROBLEMS FOR FINAL EXAM.

51

(i) Find the equation of the tangent plane to the surface at P .


(ii) Find the equation of the normal line to the surface at P .
23. Find the maximum and minimum values of the function
f (x, y) = x2 + y 2 2x
on the disc x2 + y 2 4.
24. Evaluate the iterated integral
1Z

Z
0

1x2

p
x2 + y 2 dy dx.

25. Find the volume of the solid under the surface z = 4 x2 y 2 and above the
region in the xy plane between the circles x2 + y 2 = 1 and x2 + y 2 = 4.
26. Determine whether the following vector fields are conservative or not. Find a
potential function for those which are indeed conservative.
(a) F(x, y) = (x2 + xy)i + (xy y 2 )j.
(b) F(x, y) = (3x2 y + y 2 )i + (x3 + 2xy + 3y 2 )j.
27. Evaluate the line integral
Z

(x2 + y) dx + (xy + 1) dy

where C is the curve starting at (0, 0), traveling along a line segment to (1, 2)
and then traveling along a second line segment to (0, 3).
28. Use Greens Theorem to evaluate the line integral
Z
F dr
C

where F = hy 3 + sin 2x, 2xy 2 + cos yi and C is the unit circle x2 + y 2 = 1 which
is oriented counterclockwise.
These problems are from older exams
29. (a) Express the double integral
Z Z

x2 y x dA
R

as an iterated integral and evaluate it, where R is the first quadrant region
enclosed by the curves y = 0, y = x2 and y = 2 x.
(b) Find an equivalent iterated integral expression for the double integral in
29a, where the order of integration is reserved from the order used in part
29a. (Do not evaluate this integral.)

PRACTICE PROBLEMS FOR FINAL EXAM.

52

30. Calculate the line integral


Z
F dr,
C

where F(x, y) = y 2 xi + xyj, and C is the path starting at (1, 2), moving along
a line segment to (3, 0) and then moving along a second line segment to (0, 1).
31. Evaluate the integral
Z Z
y

p
x2 + y 2 dA

+ y 2 2, 0 y x.}
E
D
32. (a) Show that the vector field F(x, y) = y1 + 2x, yx2 + 1 is conservative
by finding a potential function f (x, y).
with R the region {(x, y) : 1

x2

2
(b) Let C be the path described by the parametric curve
R r(t) = h1+2t, 1+t i
for 32a to determine the value of the line integral C F dr.

33. (a) Find the equation of the tangent plane at the point P = (1, 1 1) in the
level surface f (x, y, z) = 3x2 + xyz + z 2 = 1.
(b) Find the directional derivative of the function f (x, y, z) at P = (1, 1, 1)
in the direction of the tangent vector to the space curve r(t) = h2t2 t, t2 , t2 2t3 i
at t = 1.
34. Find the absolute maxima and minima of the function
f (x, y) = x2 2xy + 2y 2 2y.
in the region bounded by the lines x = 0, y = 0 and x + y = 7.
35. Consider the function f (x, y) = xexy . Let P be the point (1, 0).
(a) Find the rate of change of the function f at the point P in the direction
of the point (3, 2).
(b) Give a direction in terms of a unit vector (there are two possibilities) for
which the rate of change of f at P in that direction is zero.
36. (a) Find the work done be the vector field F(x, y) = hx y, xi over the circle
r(t) = hcos t, sin ti, 0 t 2.
R
(b) Use Greens Theorem to calculate the line integral C (y 2 ) dx + xy dy,
over the positively (counterclockwise) oriented closed curve defined by
x = 1, y = 1 and the coordinate axes.
37. (a) Show that the vector field F(x, y) = hx2 y, 13 x3 i is conservative and find a
function f such that F = f .
R
(b) Using the result in part a, calculate the line integral C F dr, along the
curve C which is the arc of y = x4 from (0, 0) to (2, 16).

38. Consider the surface x2 + y 2 41 z 2 = 0 and the point P (1, 2, 2 5) which lies
on the surface.
(a) Find the equation of the tangent plane to the surface at the point P .
(b) Find the equation of the normal line to the surface at the point P .

PRACTICE PROBLEMS FOR FINAL EXAM.

53

39. A flat circular plate has the shape of the region x2 + y 2 1. The plate
(including the boundary x2 + y 2 = 1) is heated so that the temperature at any
point (x, y) on the plate is given by
T (x, y) = x2 + 2y 2 x
Find the temperatures at the hottest and the coldest points on the plate,
including the boundary x2 + y 2 = 1.
40. The acceleration of a particle at any time t is given by
a(t) = h3 cos t, 3 sin t, 2i,
while its initial velocity is v(0) = h0, 3, 0i. At what times, if any are the velocity
and the acceleration of the particle orthogonal?
Spring 2009 Exam
41. For the following parts, consider the two lines
r1 (t) = h1, 1, 0i + th1, 1, 2i
r2 (s) = h2, 0, 2i + sh1, 1, 0i
a) Find the point at which the given lines intersect.
b) Find a normal vector of the plane which contains these two lines.
c) Find the cosine of the angle between the lines r1 (t) and r2 (s).
42. (a) Consider the surface S defined by the equation xy 2 + xz + z 2 = 7. Find
the equation of the tangent plane to the surface S at the point (1,1,2).
(b) Find the directional derivative of f (x, y, z) = x2 + xy 2 + z at the point
(1, 2, 3) in the direction v = h1, 2, 2i.
43. (a) Find the volume of the solid under the graph of z = 9 x2 y 2 , inside the
cylinder x2 + y 2 = 1, and above the xy-plane.
(b) Evaluate the iterated integral
Z 1Z
0

ey dy dx.

44. Determine whether or not F is a conservative vector field. If it is, find a


function f satisfying f = F.
(a) F(x, y) = h3e2y + 2xy, xe2y + x2 + y 2 i.
(b) F(x, y) = (3x2 + 2y 2 )i + (4xy + 3)j.
45. (a) Find parametric equations for the line segment C from the point (1, 5, 0)
to the point (1, 6, 4).
(b) Evaluate the line integral
Z

xz 2 dy + y dz,

where C is the line segment in part (a).

PRACTICE PROBLEMS FOR FINAL EXAM.

54

46. Let D be the region on the plane bounded by the curves y = 2x x2 and the
x-axis, and C be the positively oriented boundary curve of D. Use Greens
Theorem to evaluate the line integral
I
(xy + cos(ex )) dx + (x2 + ecos y ) dy.
C

More Problems
47. Find parametric equations for the line in which the planes 3x 6y 2z = 15
and 2x + y 2z = 5 intersect.
48. Find the equation of the plane containing the points P (1, 3, 0), Q(2, 1, 2) and
R(0, 0, 1).
49. Find all points of intersection of the parametric curve r(t) = h2t2 2, t, 1tt2 i
and the plane x + y + z = 3.
2
2
50. Find the absolute maximum and minimum of
the function f (x, y) = x +2y
2
2
2y on the closed disc x + y 5 of radius 5.

51. Evaluate

Z Z
xy dA,
R

where R is the region in the first quadrant bounded by the line y = 2x and
the parabola y = x2 .
52. Consider the vector field F(x, y) = h2xy + sin y, x2 + x cos y + 1i.
(a) Show that F(x, y) = h2xy+sin y, x2 +x cos y+1i is conservative by finding
a potential function f (x, y) for F(x, y).
R
(b) Use your answer to part a to evaluate the line integral C F dr, where C
is the arc of the parabola y = x2 going from (0, 0) to (2, 4).
53. Evaluate the line integral
Z
F dr
C

where F = hy 2 + sin x, xyi and C is the unit circle oriented counterclockwise.


R
54. Evaluate the line integral C F dr, where F = hy 2 , 2xy + xi and C is the curve
starting at (0, 0), traveling along a line segment to (2, 1) and then traveling
along a second line segment to (0, 3).
Fall 2009 Exam
55. Let f (x, y) = 3x2 /2 + 3y/2 + 2.
(a) Compute the gradient of f .
(b) Find an equation for the tangent plane P to the graph of f at the point
(1, 1, 5).
(c) Compute the distance from the plane P to the origin.

PRACTICE PROBLEMS FOR FINAL EXAM.

55

56. Let f (x, y) = 13x2 + 16xy 8x + 5y 2 6y + 5. Find and classify the critical
points of f .
57. Find the volume of the solid that lies below the graph of f (x, y) = 1 + x2 and
above the region in the first quadrant between the graphs of y = x and y = x3 .
58. The figure shows the graph of r = sin(3), the three-leaved rose. Find the area
enclosed by one loop of the graph.
59. Let r(t) = t2 i + tj + (2t3 /3 3t 1)k be the position function of a particle in
motion.
(a) Determine the velocity v(t) and acceleration a(t) of the particle.
(b) Find all times t such that the velocity is perpendicular to the acceleration.
60. Let C be the path from (0, 0) to (1, 1) along the graph of r(t) = t2 i+t3 j. (Note
that the path C is the same in (a) and (b).)
(a) Evaluate
(b) Evaluate

R
RC
C

F dr, where F = (3x2 /2 + y 2 )i + (2x2 + y)j


G dr, where G = (3x2 y + 2xy + 1)i + (x3 + x2 )j.

61. Let C be the closed path of four straight line segments obtained by starting
at (1, 0) and going successively to (0, 1), (1, 0), (0, 1), then back to (1, 0).
Compute
I
C

(sin(x) + y 3 ) dx + (3xy 2 + x + ey ) dy.

You might also like